You are on page 1of 238
Reference Guide for the Foreign Pharmacy Licensing Exam eer ireraltetong aan medication spensing pharmacy administration health care economics medicinal chemistry pharmacy aol reign pharmacy ALE LLL Aa Aad LO Lo) SHerrcieiicaertee titer rineeeietecaeesreeteeriteieeeceeeteeeeeeie ree ERE ERR REE Reference Guide for Foreign Pharmacy Licensing Exam Krisman Questions and Answers- 2nd Revised Edition TABLE OF CONTENTS SECTION-I QUESTIONS 7 SECTION-IT ANSWERS 121 SECTION-III TABLES 243 www.pharmacyexam.com 6 Reference Guide for Foreign Pharmacy Licensing Exam Krisman Questions and Answers- 2nd Revised Edition 1 Benzyl alcohol is classified as: Drug degradation 4. Drug oxidation a. Emulsifying agent b. Preservative 7. Polymorphisms generally defined as c. — Diluent a 4. Suspending agent a, Substance that may exist in more than 2. Cold cream is an example of: one crystalline form. a. Suspension b. Substance that may exist only in meta- b. — O/Wemulsion stable form, c. W/Oemulsion 4. OW/O emulsion ¢. Substance that has different viscosity time to time. 3. Egg yolk or egg white is used as: 4. Substance that reduces interfacial ten- a. Emulsifying agent sion. b. Suspending agent c.— Binder 8. The minimum concentration of a drug d. Preservative at the receptor site to initiate pharmacologi- cal action is defined as: 4, The transfer of a drug from high con- centrated areas to low concentrated areas is a. a generally defined as: b. MEC c. MTC a. Infusion a Gyan b. Levigation c. Diffusion 9. The area under curve gives useful in- 4. Dissolution formation about : 5. Which of the following is the most a ‘The amount of drug systematically ab- suitable route for administration of insulin ? sorbed. a. IM b. ‘The time to reach peak concentration. b sc c. Iv ¢. The time to reach minimum toxic 4. IV bolus concentration. 6. Noyes Whitney equation is helpful to 4. The concentration at which pharma- predict the rate of: cological actions of drug would be initiated. a. Drug diffusion b. _ Drugdissolution 10. Which of the following is the major plasma protein involved in drug binding? www.pharmacyexam.com a Reference Guide for Foreign Pharmacy Licensing Exam Questions and Answers- 2nd Revised Edition a. Globulin b. Creatinine c 4. Albumin Giycoprotein 11. Which of the following equations may be useful to find out the plasma concentration of adrug ? a Vreec S psc iy, c V,= PI, da V,=CyP 12. The initial dose of a drug through IV bolus to achieve desirable plasma concentra- tion at once is known as: Loading dose Maintenance dase Replacement dose Degradation dose 13. Which of the following is/are useful to measure glomerular filtration rate? 1. Creatinine I Inulin I, Albumin a Tonly b. Land Tonly cc. Tand IIT only a All 14. The rapid degradation of a drug by liver enzymes in a liver is defined as: Third pass effect of metabolism First pass effect of metabolism Rapid degradation Liver elimination eege www.pharmacyexam.com 15. Krisman ‘The normal renal creatinine clearance value lies between: aege 16. 200 to 300 ml/min 80 to 120 ml/min 30 to 60 ml/min 10 to 20 ml/min Which of the following is an example of an oligosaccharide? eaege Glucose Sucrose Starch Glycogen 17. Which pyrimidine base is found only in RNA? aoege Cytosine Thymine Uracil Adenine Heparin is classified as a(n): Heteropolysacharide Oligosaccharide Homopolysacharide Monosaccharide Ribonucleic acid exists in all of the following forms EXCEPT : 20. RNA mRNA qRNA tRNA Which of the following structures is 9 host for Kreb’s cycle ? a b. Mitochondria Golgi bodies Reference Guide for Foreign Pharmacy Licensing Exam Questions and Answers- 2nd Revised Edition Cytoplasmic membrane d.—_Ribosome 21. — Thesynthesis of glucose from sources other than carbohydrates is generally known a. Glycolysis b. Gluconeogenesis c. — Glycogenolysis d.— Glucogenesis 22. Which of the following amino acids should be considered an essential amino acid(s) for the body ? 1. Phenylalanine IL Leucine II. Tryptophan Tonly Tand It only Mand Il only All 23. Which of the following enzymes ca- talyses the coupling of two molecules of nucleotides to form DNA ? a. Transferase db. Ligase cc. Isomerase d. Aldehyde dehydrogenase 24. A nucleotide a building block of Sphingomide Nucleic ac Amino acid Starch 25. — Which of the following cells are in- volved with immune responses of the body? www.pharmacyexam.com Krisman 1. Blymphocytes I. T lymphocytes II. Neutrophils a. Tonly b. Land I only ¢. Wand Ionly 4 All 26. Which of the following immunoglo- bulin levels are elevated during asthma? a IgM b. IgD cE ad IgA 27 Allof the following tests are required to check sensitivity of class A weighing pre- scription balance EXCEPT : a. Armratio test b. Rider graduated beam test c. Shift test d Utest 28. The ratio of the mass of an object measured in a vacuum at specific tempera- ture to volume (in ml) of an object at the same temperature is defined as: Absolute density Specific gravity c. Relative density d. Apparent density 29, — The mean blood pressure of Mr. Ham is: 01/01/00 80 mmhg 04/04/00 90 mm hg 01/02/00 82 mmhg 01/05/00 85 mmhg 01/03/00 81.5 mm hg 01/06/00 83 mm hg Reference Guide for Foreign Pharmacy Licensing Exam ‘Questions and Answers- 2nd Revised Edition a BLS b. 85.6 c. 83.58 a 84.20 30. The deviation of data from its mean is, generally described by: The average The standard deviation The precision The accuracy eege 31. The reproducibility of results of a number of experiments is generally known as: a. Precision b. Bias c. Accuracy 4. Closelessness 32. Ifthe value of p = 0.6 in binomial dis- tribution, what is the probability of failure ? a 02 b 04 ce 03 a 10 33. The @ error is generally considered significant at: a, 1% b 3% c 53% da 10% 34. When the hypothetical value of a pa- rameter is the same as the observed value of a parameter, the error should be considered: a. Aifa-error b. —_Beta-error c. Gema-error 4. Infinitive ‘www.pharmacyexamn.com 35. Krisman Find out the degrees of freedom in a Chi-square test in a 2x2 contingency table (as- sume te gosese aege 37. s are independent) RODE ‘The F distribution generally compares: ‘Two means ‘Two variances ‘Three means ‘Three variances Which of the following elements has the highest electronegativity? eege 38. Which of the following molecules has the largest dipole movement? cH3 cl c. c CHB a FIGURE - 1 10 Reference Guide for Foreign Pharmacy Licensing Exam Questions and Answers- 2nd Revised Edition CH3 H Nea & a H CHB FIGURE-3 FIGURE-4 Fig! Fig Il Fig III FigIV 39. Which of the following molecules has the highest boiling point? a HO bo «6S c HSe 4d HCN 40. — The bond between NH, and CO, is best described as a: www.pharmacyexam.com Krisman H or So HY 4 aege 41. oO Hydrophobic interaction force Ion dipole or ion induced dipole force London force Van der walls force The process of transforming a solid directly to a vapor state is generally defined as: ee 42. Evaporation Melting Sublimation Levigation ‘The characteristic of solid substances toexhibit more than one crystalline or amor- phous form is defined as: ees 43. Isomerism Polymorphism Zwitter ion Coupling Which of the following molecules rep- resents CIS form ? \ _ Va a ~~ FIGURE -1 u Reference Guide for Foreign Pharmacy Licensing Exam Krisman Questions and Answers- 2nd Revised Edition H c. Diffusion coefficient. 4. The length of the stagnant layer. 46. Acetone is classified as a: a. Polarsolvent b. Nonpolar solvent cc. Semipolar solvent 4. Dipolar solvent 47 The process of degradation of ionic compounds intocations and anions in a pres- ence of water is defined as a. Solvation b. Hydration ©. Activation 4. Degradation 48. What happens to the solubility of al- oe cohol as the molecular weight of alcohol in- 5 creases ? b. Fig IT : a. Reduces oan b. Increases 8 c. Remain unchanged 4. Insoluble in water 44, Which of the following drugs is an angiotensin receptor antagonist ? 49. The degradation of Riboflavin by light is classified as: a. Lisinopril b. Losartan i a, Oxidation °, Methyldopa b. Reduction 4. Captopril c. Photochemical degradation 4. Racemization 45. — According to Fick’s law of diffusion, which of the following is inversely propor- 50. tionate to the rate of diffusion ? athe) deatadation af) Fence @ Procaine is highest in: a The area of the solid. a. Solution b, Thedifference between theconcentra- D»——_—Sspension tion of solute to concentration of - d. ‘Tablet solute in stagnant layer. | www.pharmacyexam.com R Reference Guide for Foreign Pharmacy Licensing Exam Questions and Answers- 2nd Revised Edition Krisman 51 Therate of oxidation is influenced by all 55. of the following EXCEPT: Tobalance the following equation, how ‘many molecules of WH CI would be required? a. Temperature (NH,),S+NICI—> NIS + NH,CL b. Radiation c. Presence of catalyst a. 1 4. Hydrolysis bo 2 co 52. — Which of the following are character- da 4 istics of pseudoplastic flow? 56. Therandom motion of solute particles 1 Viscosity of the flow generally de- creases with an increase in the rate of in colloidal dispersion is known as: shears, a. Newtonian flow b. Brownian motion Tl No yield value has been found with c. —Stoke’s law flow. 4. Non-Newtonian flow Ul. Suspension of tragacanth follows the 57. Which of the following about floccu- pseudoplastic’s flow. lated suspension is NOT true? a Toaly a. Particles of suspension form loose ag- b. Land Wonly gregates. Mand IMlonly b. Rate of sedimentation is very low. a. 1, Mand Hit only c. The time to form sediment is less. 4. ‘The sedimentation is easy to redis- 53. — Which of the following is NOT true perse. about microemulsion? 58. The rate of sedimentation is indepen- a. Themean diameter of a droplet gener- dent of : ally lies between 1010 200 nm. b Tis athermodynamically stable a. ‘The viscosity of dispersion medium. system. b. The diameter of suspended particles, © Itrequires a cosurfactant. The difference in densities between 4. Itis intermediate in property between dispersed medium and dispersed solution and emulsion. phase. ‘The lipophilic nature of particles. 54, — Asystem with considerable interaction between dispersed phase and dispersion me- dium is known as: eege Radioactive colloids www.pharmacyexam.com 59. Which of the following compounds is an acetanilide? eege CH,CONHC,H, CH,CHO. 13, Reference Guide for Foreign Pharmacy Licensing Exam ‘Questions and Answers- 2nd Revised Edition 60. The spontaneous isomerization of two stereoisomers in aqueous solution that causes specific rotation is known as: a. Zwitter ion rotation b. Micelle rotation cc. Mutarotati a. Steriorotation 61. — Which of the following is a polysac- charide? a. Dextrose b. Dextran c. Lactose d Sucrose 62. The susbstance that is isolated from the brain and produces fatty acid, galactose and sphingosine upon hydrolysis is known as: a. Sterols b. Phospholipids cc. Glycotipids a Saponins 63. — Which of the following is NOT a hy- drolyzed product of fecithins? a. Fatty acid b. Glycerol Phosphoric acid d. —_Spingosine W H CH NHCOOH www.pharmacyexam.com Krisman Which of the following is anactive moiety of the above compound ? a. Carboxylic acid b. — Imidazote c. Pyroline da Aniline 65. Albumin is an example of a: a, Simple protein b. Conjugated protein ©. Derived protein 4. — Hydrolysed protein 66. NH Fluorination of above compound will result into a well known cancer drug known as: a. Methadone ®. — 5-fluorouracil c. _ 6-mercaptopurine 4. Procainamide 67. For microbial assay of vitamin B-12, the test organism should be : a, Lieichmani bE plantarum c. Paeruginosa 4. S.pneumonia 68. Whichof the following isNOT classified as.atitrimetric method of analysis? a, Directtitration b. Gravimetric titration Complexation reaction d. Redox reaction 14 Reference Guide for Foreign Pharmacy Licensing Exam Questions and Answers- 2nd Revised Edition 69. Hemolytic anemia with abnormal he- moglobin is generally found in patients wit ‘Vitamin B-12 deficiency anemia Sickle cell anemia Iron deficiency anemia Folic acid deficiency anemia aege 70. Allof the following drugs may cause hemolysis in a patient with GOPD deficiency EXCEPT Chloroquine Sulfonamide Dimercaptrol Penicillin 71. Which of the following drugs is use- ful in a Rh negative mother with a Rh posi- tive infant ? Rho gam Octeroide acetate Immunoglobulin Pneu-immune aege 72. Which of the following is NOT true about PKU ? a. tis disease usually characterized by mental abnormalities. b. —Ahigh concentration of phenylpyruvic- acid is found in urine. ¢. Ttoccurs due to excessive secretion of Phenylalanine hydroxylase enzyme. 4. A guthrie test is normally performed to detectiit. 73. The metabolite product of epinephrine and norepinephrine is, a. Gama butyric acid b. —_Vanillymandelic acid cc. Homovanillic acid d. SHydroxyindoleacetic acid www.pharmacyexam.com Krisman 74. Which of the following is ananaerobic organism? a. L. pneumonphilia b, Cl. tetani c N. meningitis 4. E.coli 75. Which of the following organisms is responsible for causing most of the UTI? a. S. pharyngitis b. E. coli c. N. gonorthea 4. T- palladium 76. The accurate diagnostic test for a pa- tient with cystic fibrosis is : a, Mantoux test b. Sweat test c. Breath test d. Sick test 77. The allergic skin reaction character- ized by wheel formation is known as: a. Eezema b. Urticaria c. Impetigo 4. Erythema 78. — Which of the following about diabe- tes insipidus is NOT true? a. __Itisadisease usually characterized by polyuria, polydypsia, and severe thirst. b. The urine volume sometimes increases 16 to 24 liters a day. ¢. This thought to occur due to over- activity of ADH. 4. A patient should be monitored for dehydration. 15 Reference Guide for Foreign Pharmacy Licensing Exam ‘Questions and Answers- 2nd Revised Edition 719. Inwhich kinetic reaction is the rate of re- action independent from concentration ? a. First order b. Zero order c. Pseudo first order 4. Second order 80. The initial degradation of a drug by liver enzymes after oral admiaistration of the drug is known as: Enzymatic degradation First pass metabolism Relative bioavailability Fick’s degradation peop 81. Which of the following factors DOES NOT affect the protein binding of « drug ? a, Theavailability of protein for binding. b. _ Binding affinity of protein to the drug. c. The presence of competing substances for protein binding, ‘The concentration of a drug at its receptor site. 82. In which of the following conditions is an increase in plasma protein albumin found a a. Severe bums b. Cystic fibrosis c. Trauma 4. Hypothyroidism 83. Which of the following drugsis an H, se- ceptor antagonist ? Hydroxyzine Cimetidine Diphenhydramine Omeprazole aoge www.pharmacyexam.com 84, — Whichof the following drugs is indicated forreducing elevated blood concentration of ammonia in blood ? Lactulose Diphenoxylate Sucralfate Calcium polycarbophitt aoe 85. Patients with hemophilia have a defi- ciency of : RhoD AHF ADH ACE eee 86. Sodium polystyrene sulfonate is found to lower Serum K’ concentration Serum Na’ concentration Serum Al” concentration Serum Ca’? concentration pegs 87. Which of the following cells are gen- erally found to be efevated in a patient with polycythemia vera? a. Reticulocytes b. Erythrocytes c. Leukocytes d. Thrombocytes 88. Which of the following antihypertensive drugs acts by blocking alfa-7 receptors ? L Doxazosin ao ‘Terazosin TL Prazosin a Tonly b. Tand Honly Hand tf only dQ L.Tand fonly 16 Reference Guide for Foreign Pharmacy Licensing Exam Questions and Answers- 2nd Revised Edition 89. Which of the following receptor's stimu- lationprevents the release of noradrenaline? a Alpha-I receptors >» —— Beta-) receptors cc. Alpha-2 receptors d. Beta 2 receptors 90. Hypertrichosis is generally associated with the use of a. Hydralazine b. Minoxidil c. Methyldopa 4. Clonidine 91. — The preferable route for Sodium Nitroprusside is a. Intramuscular b. Oral ¢. Intravenous 4. Subcutaneous 92. — An overdose of sodium nitroprusside generally causes a, Severe hypotension b. Hypertension c. Renal failure d. Severe edema 93. The use of Sodium Nitroprusside should be strictly restricted by : a Adult men = Adult women c. Neonates d. ‘Children 94, Whichofthe following hypertensive drugs is known as aninodilator ? www.pharmacyexam.com Krisman a. Nitroglycerin b. — Milrinone c. _ Dipyridamote a Digoxin 95. — Which of the following is a Class-1A arrhythmic agent ? Lidocaine Procainamide Encainide Atenolol pege 96. Which of the following blood cho- lesterol lowering drugs is an HMG-COA in- hibitor ? a Gembifrozil b. Lovastatin cc. — Cholestyramine Niacin 97. A patient with acute hypercapnia should be treated with which of the following Doxapram Dopamine Disopyramide Ipecac 98. Which of the following drugs is found to be mucolytic or reduces the viscosity of mucous ? a. Dextromethorphan b. — Acetylcysteine cc. Terbutaline 4. Benzonatate 99. Which of the following drugsis indicated as uterine relaxant for women in labor ? Ephedrine b. ‘Terbutaline 17 Reference Guide for Foreign Pharmacy Licensing Exam Questions and Answers- 2nd Revised Edition cc. Isoetharine d.— Metaproterenol 100. Which of the following is an atropine - likedrug? Retrovir Ipratropium Carveditot Latanoprost Bese 101. Which of the following is a centrally acting muscle relaxant ? a Dantrolene b. Cyclobenzaprine ©, Bromocriptine 4. Amphetamine 102. Which of the following anti- Parkinson's drugs is a dopamine receptor ago- nist? a. Carbidopa b. — Benztropine ©. Bromocriptine 4. Amantadine 103. Which of the following diuretics acts through inhibition of carbonic anhydrase en- zyme? a. Furosemide b. Acetazolamide ©, Spironolactone 4. Hydrochlorothiazide 104. Which of the following isacommon ad- verse effect of metolazone? a Seizure b. Electrolyte loss «SLE. 4. Neuroleptic malignant syndrome www.pharmacyexam.com Krisman_ 105. Which of the following diuretics cause Ayperkalemia when used concurrently with Captopril ? EL Amitoride Tl. Spironolactone TM. — Triamterene Tonly Land Il only Mand HI only 1, 1, and IIT only eese 106. Probenecid may competitively inhibit the renal tubular secretion of : 1 Methicillin IL Methotrexate II. Dapsone a Tonly b. and Honly c. Tand If only a i, it,and ffl only 107. Which of the following can be admin- istered for treatment of insulin overdose ? I. Glucagon I. LY, Dextrose Ml. Lidocaine Tonly Tand Il only I and ITT only 1,11, and {only eose 108. Which of the following sulfonylurea agents is indicated for the treatment of diabe- tes insipidus? a, Glyburide b. —Chlorpropamide a 4. Tolbutamide 18 Reference Guide for Foreign Pharmacy Licensing Exam Questions and Answers- 2nd Revised Edition 109. The deficiency of vitamin A may cause a Osteoporosis b. —Nightblindness cc. Scurvy 4. Anemia 110. Which of the following can be used for the treatment of Methotrexate overdose ? a. Mephyton b.— Leucovorin Ca*® c. Pyridoxine a Niacin J11. Which of the following benzodiaz- epines can be safely administered to a geriat- ric patient ? a. Chlordiazepoxide b. Alprazolam .— Oxazepam 4. Prazepam 112. The active metabolite of Primidone is L Phenobarbital I. PEMA I. Trimethadione a Tonly b. Land Ionly c. and Ill only 4 I, I,and IT only 113. Which of the following drugs should not be used with Fluoxetine? a, Tranylcypromine b. Digoxin Amitriptyline d. Lidocaine www.pharmacyexam.com Krisman 114. Which of the following is not classified as an insect control chemical ? a. Insecticides b. Repellents ¢. Attractants 4. Antiseptics 115. Which of the following is NOT true about Barium sulfate? a. Itis medicinally used in roentgeno- graphy for the examination of the stomach and colon, b. _ Itisa clear solution, cc. Theprinciple adverse effectis constipation. 4d. _Ttneeds tobe mixed well with food or strained through gauze before it is administered to a patient. 116. The addition of Ascorbyl palmitate in the manufacturing process serves as a a. Preservative b. Antioxidant ©. Coloring agent 4. Flavoring agent 117. Epinephrine hydrochloride solution can be stabilized by adding a small amount of a. Sodium metabisulfite b. Sodium bisulfite c. Sulfur dioxide d. Potassium benzoate 118. Which of the following are classified as certified colors? I FDandC I. = Dand MM. External DandC 19 Reference Guide for Foreign Pharmacy Licensing Exam Questions and Answers- 2nd Revised Edition Tonly Tand Ionly Mand I only 1,1, and only ape 119. Which of the following isnot considered aprimary taste? a Saline b. Sweet ©. Spicy da. Bitter 120. A500 mg dose of a drug administered via L.V. injection produces a plasma concentra- tion of 2.5 meg/ml after 16 hours. If the initial plasma concentration of the drug is 10 meg/ml, bioavailability is 1, and volume of distribution is 120,000 L, what is the half-life of the drug? a 2hours: b. Shours c.— Shours d.15hours 121. Sweet taste of acompound is generally attributed to: Presence of Htions Presence of Oi ions Presence of cations and anions Presence of alkaloids eese 122. The alcohol content of low iso-alco- ho) elixir is : a 8to 10% b. 1510.23 % c. 501080 % a 7310 78% 123. Erythema multiform is generally de- scribed as: www.pharmacyexam.com Krisman a, Thepresence of erythematous macules and papules. b. The presence of hair on skin. ©. ‘The presence of large flaccid bullae on skin, 4. The presence of scaling and sloughing on entire skin. 124. The antidote for Acetaminophen tox- icity is: a EDTA b. — Neacetyleysteine cc. Mesna diazepam 125. Which of the following drugs may cause cholestatic jaundice? I. Chlorpromazine I Erythromycin estolate TI Indomethacin Lonly Land Il only Mand III only 1, IL, and III only peer 126. The principal adverse effect of Clinda- ARF (acute renal failure) TEN (toxic epidermal necrolysis) (antibiotic associated colitis) ADR (adverse drug reaction) 127, Whichof the following isa majoradverse effect of Chloramphenicol? a. ‘Thrombocytopenia b. Aplastic anemia . Hemolyticanemia 4. Agranulocytosis Reference Guide for Foreign Pharmacy Licensing Exam Questions and Answers- 2nd Revised Edition 128. Which of the following drugs mayNOT cause hemolysis in patients GePD deficiency? a Quinine b. — Sulfonamide c. _ Nitrofurantoin 4. Erythromycin 129. Ringing or buzzing in the ear is asso- ciated with : a, Streptomycin b. Salicylate c. Methyldopa 4. Minocycline 130. Pigmented retinopathy is highly asso- ciated with a. Chlorpromazine b. — Thioridazine ec. Clozaril 4. Haloperidol 131. Which of the following drugs is asso- ciated with pulmonary dysfunction ? 1. Bleomycin I. Amiodarone MI, Nitrofurantoin a Tonly b. Land Ionly cc. and If only dM, and [ff only 132. Which of the following phase trials includes extensive clinical trials ina human? Phase Ifonly Tl Phase Ilonly TI. Phase Tonly www.pharmacyexam.com a Tonly b. —_TandIonly Hand Ilfonly 4. L.I,and only 133. When structurally different chemicals produce the same clinical results, itis generally defined as: ‘Chemical equivalence Bioequivalence ‘Therapeutic equivalence Clinical equivalence aoge 134, The parameter that is generally NOT included in evaluating the bioegivalency of ‘two or more formulations of the same drug is Peak height concentration. Concentration at receptor site. Time to reach peak concentration. AUC. apse 135. A substance that kills microorganisms but not bacterial spores is generally defined as: a. Bactericide b. Sterility ce. Germicide d. Disinfection 136. How many grams of Heparin is re- quired to prepare I quart of 0.45% solution ? 2.21 grams 5.56 grams 4,32 grams 3.15 grams 137. Which of the following about moist heat sterilizationis NOT TRUE? a Reference Guide for Foreign Pharmacy Licensing Exam Questions and Answers- 2nd Revised Edition a. Ttisawidely used method forsteril- ization of mineral oil, greases and waxes. b. The cause of death of organisms is generally attributed to coagulation of cellular proteins of organisms. ©. The substance should be kept loose to allow direct penetration of steam da The substance should be kept at least 15 minutes under pressure steam at a minimum of 121° temperature. 138. Which of the following gases are gen- erally implied forsterilization of pharmaceu- ticals? 1. Ethylene oxide Il Formaldehyde II. Sulfur dioxide Lonly Land I only Mand III only 1, fl, and II only pee 139 How many cc of 75% alcohol is re- quired to prepare 35% of 500 cc of alcohol ? (You have 10% of 1000 ce of alcohol in stock.) a 192.30ce b.— 200ce c — 450ce d 325.55ce 140. How many tablets (approximately) of 0.125 mg Lanoxin is required if the measured plasma concentration of drug is 1 meg/ml and the desired plasma concentration of drugiis 1.5 meg/ ml. The apparent volume of digoxin is (0L/Kg fora 70 kg patient. (Assume S and F= 1) a One b. Four ce. Three d Two www.pharmacyexam.com Krisman 441. Sterilization by U.V. radiation generally requires a wavelength of: a 320 nm b 253 nm e150 nm ad 475 nm 142. [If 1 teaspoon of Thioridazine intense solution (30 mg/cc) is diluted up to the 480 cc. mark with plain water, what would be the strength of the drug in mg/m in the final so- lution ? 1 mg /ee 0.52 mg fee 0.31 mglec 0.75 mg/ec eege 143. Which of the following tests are used to identify Laminar flow efficiency and qual- ity of sterilization? 1. Smoke test IL Doptest IL. Microbial test Tonly Tand If only Tand Hil only 1,1, and IIT only 144... The rate of hydrolysis depends on L Pressure I, pHoofthe solution 1. Temperature Tonly Land I only Mand only i, {and II only aege 22 Reference Guide for Foreign Pharmacy Licensing Exam Questions and Answers- 2nd Revised E A485. If 1000 tablets of Risperdal 1 mg cost ‘\— $2250, and the mark-up on prescriptions is 20%, what would be the retail price of 30 tablets ? a $150 b. SIT ce. $500 da $81 146. NH2 1H2 Ve — Q COOH ‘OH (P-Aminosalicylic acid) (m-aminophenol) The formation of m-aminophenol occurs due to : a. Oxidation of p-aminosalicylic acid. b. _Decarboxylation of p-aminosalicylic acid. c. Reduction of p-aminosalicylic acid. 4. Hydrolysis of p-aminosalicylic acid 147. According to the International Phar- maceutical Federation, the maximum percent- age of overages is limited 0 : 10% of over the labeled potency. 20% of over the labeled potency. 30% of over the labeled potency. 40% of over the labeled potency. pers 148. At what concentration doessucrose re- tard the growth of microorganisms. = 20% wiw b. 30% wiw 65% wiw d 85% wiw www.pharmacyexam.com Krisman 149. Which of the following best describes the Ingram’s regimen ? ‘Combination of anthralin + UVA Combination of coaltar + UVB Combination of anthealin + UVB ‘Combination of psoralen + UVA. pegs 150. Which of the following drugs is indi- cated for treatment of eystic fibrosis? Domase Alfa Calcitonin salmon Acetazolamide Methotrexate aoge 151. Find out the ratio of ionized to union- ized species of a drug at pH = 7. The pKa of the drug is 5 1 50:1 25:1 200:1 aege 152. Ocusert Pilo 20 generally delivers a. 20 meg of Pilocarpine per hour for 7 days. b. 20 mg of Pitocarpine per hour for 7 days. ¢. 20 meg of Pilocarpine per hour for3 days. 4. 20 mg of Pilocarpine per hour for 3 days. 153. Which of the following antiglaucoma medications has aCabonic Anhydrase Enzyme inhibition property? a. Timo b. — Dipiveftine 23 EEE rrr Tl lv lCrrGrvr Fr --; “ Reference Guide for Foreign Pharmacy Licensing Exam Questions and Answers- 2nd Revised Edition ¢. Dorzolamide 4. —_Latanoprost 4154, If a dropperiscalibrated to deliver 325 mg of iron sulfate per 0.6 cc, and the adult dose of the drug is 325 mg, what would be the dose of drug in ce required for a 15 month old infant . a Lec b. 03 cc c. 0.06 cc d 0.01 ce ASS. Which of the following is/are true about Triphasic oral contraceptives? 1 Triphasic affects the follicular, ovula- tion and luteal phases of the menstrual cycle successfully and provides more favorable effects than biphasic and monophasic. TI tis generally formulated with Jow progesterone content. TI. The principal advantage associated with Triphasic is the uniform direction for taking the pills. Tony Land II only Mandl only 1, and Rt only pegs 156. Which of the following phase(s) is/are generally dominated by estrogen ? 1. Follicularphase I Ovulatory phase IL. Luteral phase a. Tonly b. — TandIlonly www.pharmacyexam.com Krisman c. MandIlLonly dL Mand Ht onty 157. Which of the following about Metabolic Acidosisis NOT TRUE ? a, It generally occurs due to loss of bi- carbonate frome the body. b. _ Hypokalemia is generally a result of metabolic acidosis. ¢. Administration of Arginine HCl gen- erally resolves metabolic acidosis. 4, It stimulates the respiratory center to increase excretion of CO, from the body. 158. Which of the following ingredients acts as a contact poison that disturbs the Parasite’s nervous system? a, Lindane b, —_Permethrine c.—Pyrethrin 4. Pyrimethamine 159, Which of the following is a cough ex- pectorant? a Guaifenesin b ——Dextromethorphan ©. Benzonatate 4. Diphenhydramine 160. HbA, 6% value indicates : a 80:mg/dl blood glucose b. 120 mg/dl blood glucose ¢. 360 mg/dl blood glucose d. 160 mg/dl blood glucose Reference Guide for Foreign Pharmacy Licensing Exam. ‘Questions and Answers- 2nd Revised Edi 161. How many milliequivalents of Ferrous U suifate are present in 325 mg of Ferrous sul- fate 2(Molecular weight = 151.85 gm/mole) 5.6meq 10.12 meq 7.5 meq 4.28 meq aoge 162. Which of the following adverse effect(s) is/are associated with insulin therapy? I. Hypoglycemia 1 Lipoatrophy IIL. —_Lipohypertsophy Tonly Tand II only and III only I, Hand III only pose 163. Acarbose is contraindicated for pa- tients suffering from I. _ Inflammatory bowel disease Wi. Patients with colonic ulceration III. Patients with intestinal obstruction a Tonly b. Land ILonly c— Tand Ill only dé. I, Iand Ill only 164. Which of the following isNOT asymp- tomof hypoglycemia? Confusion Bradycardia Difficulty inconcentration ‘Sweating aege 165. Whichofthe following diabetic drugs pro- duces disulfiram reaction with alcohol? www.pharmacyexam.com Krisman a. Acarbose b. _ Chlorpropamide cc. Metformin d. ——_Repaglinide 166. Normal threshold value for glucose inthe kidney is a. 100 mg/dl b. 120 mg/dl cc. 180 mg/dl d 220 mg/dl 167. How many mEqs of Na’ are present in \/ 0.9% 250 cc solution of NaCl ? (equivalent weight of NaCl = 58.5] 50.5. mEq 38.5 mEq 225 meq 58.5 mEq aes 168. Which of the following is NOT rec- ommended to treat hypoglycemia induced by Acarbose ? a Dextrose b. Table sugar ©. Glucagon d. Glucose 169. Whatis the half-life of a drug that has a rate constant of 0.067 days” in first order kinetic ? a Sdays b.-7.23days ©. 10.34 days 4. 12.51 days 170. Which of the following isNOT TRUE, about Vancomycin HCl ? 25 Reference Guide for Foreign Pharmacy Licensing Exam Krisman Questions and Answers- 2nd Revised Edition a, Nephrotoxicity and ototoxicity are common complications of therapy. 174, Which of the following thyroid drugs should in pregnant women avoid ? b, TV Vancomycin is indicated for treat- a. Propylthiouracil ‘ment of amtibiotic associated colitis, b. — Methimazole c. _ Whentaken orally, itis poorly cc. Propranolol absorbed from the GIT. 4. Levothyroxine d, _Itis available in oral solution, pulvules 175. Which of the following statements about and injection form 171. Which of the following drugs should patients with hypertensensitivity to Me- salamine avoid ? thyroid supplements is/are true? L Livothyronine is associated with head- ache, palpitation, tremor and diarrhea, and is fess recommended for treatment of hypothyroidism. 1. Sulfasalazine TL Olsalazine Il. _Desiccated thyroid preparations have TI. Mesalamine a variable T3 to T4 ration and are less recommended by physicians. a Tonly b. — Tand Tonly Ill. Levothyroxine is the most recom- c. Mand If only mended thyroid supplements. d. 1, fand If only a Tonly 172. Which of the fottowing is NOT a sign b. Tand Il only of Grave's disease? cand If only 4 1 Wand tif oniy a. Fast heartbeat b. Weight gain 176. Which of the following drugs is indi- c. Soft skin cated in smoking cessation programs ? 4. High basal metabolic rate a. Bupropion 173. Which of the following is/are caused b. ‘Tramadol by thyroid hormone deficiency ? cc. Tamsulosin d. Risperidone 1. Myxedema U. — Cretinism WL Grave's disease 177, Which of the fotiowing drugs inhibits the aggregation of platelets ? a Tonly L Ticlopidine HCL. 6. and only T. Aspirin ¢. Mand If only II. Clopidroge! 4 I. MandHtonly www.pharmacyexam.com Reference Guide for Foreign Pharmacy Licensing Exam Questions and Answers- 2nd Revised Edition Tonly Tand Monly Mand Ill only 1, Hand III only pege 178. Which of the following drugs mobilizes bone calcium into blood ? a b. c. Calcitonin Salmon a. Alendronate Na 179. Diphydrotachysterol is indicated for treatment of 1 Tetany I Idiopathic Tetany II. Hypoparathyroi a Tonly b. Land Ionly c. Hand Ionly aI, IMand Ill only 180. Which of the following is NOT a sign of Morphine overdose? Cold and clammy skin a b. Respiratory depression c. Pupil dilation 4. Convuisions _AB1._ How long will it take to decompose Lanoxin in elixir to one half of its original concentration in an order that is first order kinetic ? [ K = 0.023 min] a. 40.45 minutes b. 50.60 minutes c. 75.80 minutes d. 30.13 minutes 182. Which of the following glasses are most chemically resistantor least leachable? ‘www.pharmacyexam.com Krisman a. TypeI borosilicate glass b. Type II sodalime treated glass c. Type III sodalime glass d.—TypelVglass 183. According to Federal standard, class 100 clean room is generally defined as : a. Not more than 100 people working in aroom. b. Not more than 100 particles per cubic ft of 0.5 mem or larger size. c, Not more than 100 bacteria per cubic ft of 40 mem or larger size. Not more than 100 viruses per cubic ft of 40 mem or larger size. 184, Which of the following isNOT TRUE about a HEPA filter ? a, ___Itis generally described as a high effi- ciency particulate air filter. b. _Ithasanefficiency of removing 100% of particles 0.3 mem or larger. c. A Dioctyl Phthalate test (DOP) generally finds out the efficiency of HEPA filters. d, __Itisuseful forthe preparation of parenteral products, 185. Cold sterilization of parenteral solution. isdefined as: a, Removing 2 mem or larger par- ticles from the solution. b, Removing 0.2 mem or larger par- ticles, including microorganisms, from the solution. 27 Reference Guide for Foreign Pharmacy Licensing Exam Questions and Answers- 2nd Revised Edition ¢. Sterilization of the parenteral solution with hot and cold compresses. 4. Sterilization ofthe parenteral solution by freezing the solution below freezing point 186. Lyophilization is defined as: a. The removal of water by method of sublimation from the product after it is frozen, b, Direct evaporation of a substance from its solid form. c. Preparation of colloidal solution. 4. Removal of particulate matter by fil- tration 187. Which of the following is NOT de- scribed as aquality assurance and control test for parenieral solution? a. Sterility test b. Particulate matter evaluation c. Pyrogen test 4. Schilling test 188. Which of the following is NOT anon verbal communication? a. Body posture b. Facial expression c. Open-ended question 4. Distance of the patient 189. Staff model HMO organizations gen- erally own: 1 Healthcare facilities I. Employed physicians Hf, _ Nonemployed physicians www.pharmacyexam.com a. Tony b land Il only cc. Hand!lTonly 4 [Mandl only 190, The disadvantage associated with mail- order pharmacy services are I. Alackof pharmacist counseling. TL _Alack of review of patient's pro- file. IIL. Theuse of recycled prescription drugs. Lonly Land I only Hand IIL only 1, Hand Ill only pee 191. Which of the following best describes the prospective payment system? a. Reimbursement to the hospital based on cost incurred during hospitalization, b. Reimbursement to the hospital in ad- vance, with a specific rate based upon diagnosis of patient. c. Reimbursement to hospitai by patient when he/she is discharged, 4. Prescription purchased by patient by paying cashs. 192, Most hospital pharmacies generally order their products from one wholesaler with a small percentage fee. This system is generally knownas: Floor charge system Direct charge system Prime Vendor system Indirect charge system ee ee Reference Guide for Foreign Pharmacy Licensing Exam Krisman Questions and Answers- 2nd Revised Edition 193. Which ofthe following isNOT true about 197. Ifthe pricing of prescription counts based a free floor system ? on the percent mark up, what would be the dispensing price of a drug that costs $20.00? a. Ithasapredetermined list of medica- (assume the % markup is 50%) tions. b. The charges of used medications are a $10 snade to a patient upon discharge from b $25 the hospital. c $30 ¢. __Ithas prepackaged medications in a S5 standardized containers. , 198. Ifthe pharmacy has been set to receive 4. The inventory is generally taken by a flat professional fee independent of the cost pharmacy personnel on the spot. of the ingredient, what would be the dispens- ing price of a drug witha $6,00 professional fee? 194, Which of the following is NOT a vo- (assume the cost of the ingredient is $65) cal quality ? a 105 a Pitch bo o71 b. Gaze 3250 c. Tone dG a Range 199. Which of the following is generally 195. Which of the following questions gen- classified as primary literature? erally requiresan answer in the form of ‘yes’ or ‘no"? a. Factsand comparisons b. Index medicus a, Closed-ended questions ¢. Matiandale extra pharmacopeia b. Open-ended questions d. Pharmacy times €. Indirect questions d. Multiple questions 200. Which of the following is NOT an important consideration when selecting arti 196. One of Mr. Jacob’s prescription signas eee i eater ead Indicates “1 gtt ou gid x10." It can be interpreted as: a Lagtime b. Cost a. 1 drop toright eye four times a day for c. Selectivity of indexing 10 days. 4. Theauthorofanarticle - 1” b. Apply 1 tube to affected area four times a day for 10days. 201. Which of the following place is more ¢, I drop to both eyes four times a day prone to accidental poisoning in children in for 10 days. the late morning? 4. Idrop to lefteye four times a day forl0 days, a. Kitchen www.pharmacyexam.com 29 Reference Guide for Foreign Pharmacy Licensing Exam Questions and Answers- 2nd Revised Edition b. Garage Bathroom 4 Bedroom 202. Pesticides and petroleum products acci- dental poisoning are more likely to occur where? a. Kitchen b. Garage cc. Bathroom 4. Bedroom 203, Which of the following factors gener- ally increases the risk of poisoning? L Age IL Accessibility IIL. Type of container Tonly Land Il only and III only 1, {fand I only Boge 204. Which of the following controlled sub- stance schedules has a high potential for abuse? a ‘Schedule b. Schedule II c Schedule [IT d. Schedule IV 205. Which of the following drugs is exempt from the poison prevention packaging act? a ‘Sulfasalazine b. Sublingual nitroglycerin cc. Hydrochlorothiazide 4. Glyburide 206. Which of the following drug(s) shouid be dispensed with PPI? www.pharmacyexam,com Krisman 1 Isotretinoin a Loovral UL. Loestrin Fe a Tonly b. — TandIlonly Mand ffonly d. I, Hand II only 207. Jacob Bhuce wrote a prescription for Methylphenidate 5 mg, Which of the follow- ing DEA numbers proves the authenticity of Mr, Jacob Bruce as a physician? a AA 4583215 b. AB 3245671 ce. BA2354321 d BY 5213215 208. Which of the following about dispens- ing controlled substances V without a prescrip- tion by a pharmacist is NOT true? a. Donotdispense more than 240 cc or 48 solid dosage units of any controlled substances containing opium or more than 120 cc of other controlled sub- stances to the same purchaser in any given 48 hours period without a pre- scription. b. The purchaser should be at least 21 years of age. c. A bound red book should be main- tained with the name and address of the purchaser, name and quantity of the controlled substance sold, and pharma cist initials with the date the drug was dispensed. Dispensing should be done only by the pharmacist. Reference Guide for Foreign Pharmacy Licensing Exam Questions and Answers- 2nd Revised Edition 209. ‘The partial filling of schedule II controlled substances must be finished within a. 24 hours of initial filling b. 48 hours of initial filling c. 72 hoursofinitial filling d. 96 hours of initial filling 210. Mr. Moose has the following values for Y his blood pressure in mm Hg. Mon 282 Tues 262 Wed 275 ‘Thurs. 285 Fri 290 What is the mean blood pressure for Mr. Moose? a B12 b. 283.4 c. 290.1 d. 278.8 _211. Findoutthe volume of Nitic oxide at OC and 760 mm Hg. The volume of Nitric oxide at 25°C temp and 745 mm Hg is found to be 45mil (assume that the gas is behaving ideally) a, 100.25 b.25.10 ce 40.41 60.65 212. A system (one component) with gas, liquid and solid has a degree of freedom that is: a. Trivariant b. —_Bivariant ¢. Invariant d. Univariant www.pharmacyexam.com Krisman 213. Which of the following drugs should be classified as aca-channel blocker ? a. Amlodipine b. Acyclovir c. Losartan 4. Epotein 214, Thepart ofthe molecule that absorbs the ultraviolet light or visible light is known as: a. Thixotropy b. Chromophores cc. Protogenic 4. Turbidity 215. 1gram molecular weight of solute in 1 liter of solution is generally defined as : a. Molarity b. — Molality c. Normality 4. Mole fraction 246. Find out the molarity of a solution of sodium bicarbonate. An aqueous solution was prepared by adding 60 gm of sodium bicar- bonate into1000 ml of water. [Mol weight 84.gm] a -0.714M b. — 0.102M c 05M da. 03M 217. Find out the equivalent weight of Ca(PO,),? [MW of Ca,PO,), = 310 gm/mole] a. 80gm/mole b. 51.7 gm/mole cc. 40gm/mole 4. 35gm/mole 31 Reference Guide for Foreign Pharmacy Licensing Exam Questions and Answers- 2nd Revised Edition 218. When two compartments are separated by asemipermeable membrane and only sot- vent molecules can pass from one compartment (other compartment, the process is defined as: a Diffusion b. Osmosis ©. Dispersion d. Suspension 219. ‘The conductance of a solution contain- ing 1 gm of equivalent weight of the solute measured in a cell when both electrodes are spaced | cm apart is known as: a. Molarconductance b. Creatinine clearance c. Equivalent conductance a. Molal conductance 220. Find out the concentration of H' ions, if the pH of the solution is 5. a. 10° b. 10" c. 108 a 10 221. Find out the pKb of a solution having pKa3.s. 15 10.5 14 1 eese 222. What would be the log [salt/acid] ratio for an Acetic acid solution with a pH of 6 and pKa of 4.76? a. 1.00 b 240 ce 124 “350 www.pharmacyexam.com Krisman 223. What is amaximum buffer capacity of an acetate buffer witha 0.04 molefit concentra- tion? a 0.0230 b 6.0510 c. 0.0411 6.9323 224, The increase in mutual solubility of two partially miscible liquids by the addition. of a third substance is defined a: Catalyst Blending Promoter Levigation eege 225, Which of the following compounds is a naturally occurring chelate ? EDTA Hemogtobin Ascorbic acid Cus ion nose 226, The degradation of Aspirin to salicylicacid and acetic acid in the presence of water is generally knownas: a. Oxidation b. Hydrolysis c.—Photolysis 4 Epimerization 227. Which of the following isNOT TRUE about Lyophilic colloids? a. The disperse phase of his type of col- toid generally consists of targe or- ganic molecules. b. Molecules of the disperse phase have ahigh affinity for dispersion medium, 32 Reference Guide for Foreign Pharmacy Licensing Exam Questions and Answers- 2nd Revised Edition Dispersed particles can be easily precipitated out in the presence of small concentration of electrolytes. 4. Molecules of the disperse phase spontaneously form the colloidal solution. 228, The range of particle size of colloidal dispersion is: Less than | micrometer 0.5 to 1.0 nanomicron Greater than 0.5 micrometer 1.0 mem to 2.0 mem eese 229. Colloidal mercury is used as a diag- nosis agent for a. Lyme disease b. Syphilis c. Pneumonia d. Rabies 230. ‘The Brownian motion of colloidal par- ticles can be reduced by increasing the: Surface area of colloidal particles. Viscosity of dispersion medium. Molecular weight of particles Interfacial tension between disperse phase and disperse medium. eee 231. The protective property of colloid is ‘generally expressed by a: a. Red number b. Gold number c. Silver number 4. Green number 232. A sample of sodium bicarbonate powder ~ weighing 150gm was found to have a bulk vol- ume of 90cm! when placed in a 100 ml gradu- ated cylinder. Calculate the porosity of the pow der. (Density of NaHCO, = 3.5) www.pharmacyexam.com Krisman a 65% b. 52% 15% a 85% 233. Which of the following factors does NOT affect the flow property of particles ? a. Porosity b Shape ce. Color 4. Density 234, Which of the following isNOT TRUE about plastic flow? a. When shear stress exceeds the yield value, the flow acts as a Newtonian flow. b. _ Themoreflocculated the suspension, the lower the yield value would be. ¢. The flow curve generally starts at a particular point referred to as yield value. d. The substance that exhibit a yield value is classified as solid according tothis flow. 235. If the concentration of reactant A is double in a reaction that is second order inA, by what factor will the rate of reaction change? eege mane 10. 236. Whichof the following substances exhibits the pseudoplastic flow? 3 Reference Guide for Foreign Pharmacy Licensing Exam Questions and Answers- 2nd Revised Edition a. SMZ/TMP suspension b. Sodium carboxymethylceltulose cc. Suspension of ZNO 75% 4.49% alcohol solution 237. Which of the following systems is gen- erally recognized as a “shear thickening” system? a. Plastic flow b. _Pseudoplastic flow c. _ Dilatant d.— Thixotropy 238. The property of flow that generally con- verts toasolution from a gel upon applying stress, and converts back to a gel from a sol upon resting is known as: a Plastic flow ’. — Thixotropy c. _ Pseudoplastic flow 4. Dilatant 239, The principal of thixotropy is gener- ally applied in all of the following formulations EXCEPT: Callhydryl lotion SMZ/TMP suspension Peniciitin G Procaine depot injection Lanoxinelixir Bese 240. Which of the following factors doesNOT affect the sedimentation of flocculated particles? a Size of flocculated particles. Porosity between sediment particles. ¢. Compacting and rearrangement be- tween sediment particles. d. Shape of floccutated particles. 241. Which of the following isNOT catego- rized as an instability of emulsion? www.pharmacyexam.com Krisman a Creaming b. — Microemulsion c. Breaking 4. Phase inversion 242. Which of the following is NOT true about microemulsion? a. Unlike emulsion, it appears as @ clear solution. b. They are thermodynamically stable. c. _ Thediameters of droplets are between 10 to 200 nm. d. Itis generally helpful to solubilize drugs in a pharmaceutical system. 243, The rate of release of drug from aGas- trointestinal Therapeutic System (GITS) fol lows: a. First order b. Zero order c. Second order d. Pseudo first order 244, Medically indigent means medical ‘expenses are usually covered by a: a. Medicare program b. Medicaid program c. — Selfcare program d. Blue Cross Biue Shield program 245. Which of the following about Retro- spective payment is NOT true? a, It generally depends on hospital costs and hospital charges. b. The payment increases as the costs of the hospital increase. Reference Guide for Foreign Pharmacy Licensing Exam Questions and Answers- 2nd Revised Edition ¢. Its most preferred by third party insurances. 4. The amount of payment generally de- pends on the actual service provided by the hospital during the patient’s stay. 246. Which of the following generally de- scribes the prospective payment system? 1. Diagnostic related group services. (DRG) Tl. Predetermined rate of per day costs of hospital I. Predetermined rate per episode of ill- ness, Tonly Tand Honly Mand Ill only I, ILand If only eege 247. Which of the following about Preferred Provider Organizations is NOT TRUE? a, This program includes groups of hos- pitals and physicians that are eady to provide services toemployers, insurance providents, or other third party carriers on the basis of fee for services. b. The subscriber can negotiate the price of fees. The freedom of choice is generally limited to a number of hospitals and physicians. 4. This less accepted by providers due to scounted fees and pressure to pro- Vide services at reduced costs. www.pharmacyexam.com Krisman, 248. Which of the following is the most commonly used prescription pricing system? Dispensing Fees method Percentage Markup method ‘The Per diem charge method Cash method Bese 249, Which of the following is NOT a part of Retrospective DUR study ? Number of drugs per patient. Number of doses of drugs per patient. Cost of drugs per patient. Floor stock system of drug . pegs 250. Which of the following has the largest and most current information sources ? 1. Primary literature Tl, Secondary literature IL, Tertiary literature only Jand Il only Mand I only 1, and IM only Bese 251. Whichofthe following committees should review the reported adverse drug reaction (ADR)? FDA Pand T committee FTC EPO pose 252. Which of the following is anonparamet- ric statistic? Mean ttest Chi-square test Standard dey aeoe 35 Reference Guide for Foreign Pharmacy Licensing Exam ‘Questions and Answers- 2nd Revised Edition 253. Which of the following tests is used to check for significance between wo or more ‘groups of means? a. Chi-square b, tests cc. Analysis of variance 4. Regression analysis 254. Which of the following error(s) is/are ‘commonly associated with compliance? 1. Overutilization of dmg tL Underutilization of drug IIL. Administration of medication at inap- propriate times. a Lonly b. Jad only c. Mand Ill only 4. [Mand If only 255. Which of the following factors are a re- sultin non compliance of prescribed medications? 1 Ageover 65. I Drug induces side effects. Ml _Fearof drug dependency. a Tonly b. Land I only c. Mand IMfonly 4 1 Hand (only 256. Thehypersensitivity reaction that gener- ally occurs due to pollen or grasses is classified = Wer jeveh ‘Type hypersensitivity reaction ‘Type Ihypersensitivity reaction ‘Type I hypersensitivity reaction ‘Type IV hypersensitivity reaction pege ‘www.pharmacyexam.com 257. Which of the following drugs is indicated for the treatment of Prcarini.Pneumonia ? a. Pentamidine b. Erythromycin c. Phenobarbital 4. Gabapentin 258. ‘The principal adverse effect of Cloz- a. Hypotension &. _Hypertrichosis c. — Agranulocytosis a. Renal failure 259. Which of the following isNOT aType III hypersensitivity reaction ? a Rheumatoid arthritis Lupus erythematosus c. Contact dermatitis 4. Sjogren syndrome 260. Grave's diseases generally classified as: ‘Type I hypersensitivity ‘Type V hypersensitivity ‘Type It hypersensitivity ‘Type III hypersensitivity ees 261. Which of the following isNOT an organ specific autoimmune disorder? a. Hashimoto's thyroiditis b. Myasthenia gravies ©. SLE. d. Multiple sclerosis 262. Whichof the following about monoclonal antibodies is/are true? TL. _ Monocional antibodies are a specific antigen derived froma single cell line, Reference Guide for Foreign Pharmacy Licensing Exam Questions and Answers- 2nd Revised Edition Tl. —_Murine monoclonal antibody and Digibind are the only two FDA approved monoclonal antibodies available in market Il, They are much safer and specific ‘compared to regular antibodies. a Tonly b. Land If only cand Ill only 4. I, Mand If only 263. Which of the following drugsis classified asa Loop diuretic ? a. Furosemide b. Spironolactone c. Acetazolamide a Mannitol 264. Which of the following antihypertensive agents acts by stimulation ofalpha-2 receptors? a, Hydralazine b. Methyldopa cc. Captopril d. Propranolol 265. Which of the followings true about the transfer of a drug from the stomach into the blood stream? 1. Therate of passive diffusion of the drug generally depends on the lipid solubility of the drug. IL Thedrug generally crosses the cell barrier by passive or active transfer mechanism. Il Thehigher the amoun‘ of ionized fraction of drug, the higher the amount of drug will transfer and be absorbed into the blood. ‘www.pharmacyexam.com Krisman only Tand I only and III only 1, and Ii} only poge 266. Basic drugs are found to bind to: 1 Albumin I Beta-globulin IIL. Acid glycoprotein a Tonly b. Land Ionly c. Hand Ilonly 4 I, Mand Il only 267. Which of the following indicates the phase II drug metabolism? Oxidation Reduction Glucuronidation conjugation Hydrolysis gree The percussor for 5-HT is : Tryptophan Leucine Isoleucine Valine eege 269. Which of the following is NOT a phar- macological action of Histamine? a. Cardiac depression b. _ Gastricacid stimulation a ‘Vasodilation 4. Contraction of smooth muscles 270. Which of the following prostaglandins have agastric mucosal protection property? a, PGH, b. PGI, 37 Reference Guide for Foreign Pharmacy Licensing Exam ‘Questions and Answers- nd Revised Edition © PGE, 4. PGD, 271. Which of the following substances is responsible forcausing dry cough in patients receiving ace-inhibitor therapy? a. Prostaglandin b.—Bradykinin © Interleukine Leukotriene 272. Which ofthe following isanimportantrisk factor associated with atherosclerosis? a LDL b. HDL | viby d.— Thiglyceride 273. Which of the following factors associates with aggregation of platelets ? a. Leukotriene b. Thromboxane A, c. Bradykinin d. Arachidonic acid 274. Which of the following is a muscar- inic receptor antagonist a. Albuterol b. Ipratropium c. Theophylline 4. Triamcinolone 275. Metabolic alkalosis is reported with : a. Spironolactone b. —Hydrochlorothiazide ©, Amiloride 4. Thamterene www.pharmacyexam.com Krisman 276. Which of the followingreceptor(s) is/are associated with emesis ? 1 H, receptors I. D, receptors ML, SHT, receptors Tonly Tand IT only Mand Ill only 1,Mand I only pBege 277. X24 ¥! = I would be graphed as : Parabola Circle Straight line Belipse aes 278. Theequation below shows the formation of ethanol and acetic acid : CH,COOC,H, + OF (high concentration) —_— CH,CHOOH + C,H,0H The above equation follows: a First order b. Zero order C Pseudo first order 4 Second order 279. IfY=1-X*then calculate the limit when 38 Reference Guide for Foreign Pharmacy Licensing Exam ‘Questions and Answers- 2nd Revised Edition 280. Which of the following is classified asa negative catalyst ? L Ascorbic acid Il Sodium sulfite II Sodium metabisulfite a Tonly b. and Ionly c.— Tand If only 4 I, Mand If only 281. Which ofthe following drugs isan inhibi- tor of HMG-COA ? a. Gembifrozil b. Simvastatin cc. Theophylline d Niacin 282. Which of the following glands generally matures the T cell ? ‘Thyroid gland ‘Thymus gland Pituitary gland Adrenal gland ae ge 283. Which of the following is a drug of choice for treatment of multiple sclerosis? a. Cyclosporine b.— Dantrolene ©. Methylprednisolone 4. Cyclophosphamide 284. Which of the following is NOT an example of a live attenuated vaccine ? a. Measles b. Diphtheria Rubella 4. Mumps www.pharmacyexam.com Krisman 285. Whichof the followingisNOT tue about acid test ratio? a, It generally indicates liquidity of a pharmacy. b. Iti expressed by the ratio of quick asset to current liability of a pharmacy. c. Fora pharmacy’s successful financial position, this ratio would generally be greater than 2. dd. Thecloser the current liability toa pharmacy’s quick asset, the more ideal the acid-test ratio. 286. Working capital is generally expressed a. Inventory ratio to working capital, b. _ Thedifference between current asset and current liabilities of pharmacy. ©. Quick assets ratio to current liability. d. Cost of goods ratio to inventory. 287. Which of the followingisNOT true about exotoxin? a. Itis generally an end product of ‘metabolism produced by gram +ve bacteria, b, Iti water soluble and has the ability to pass through a surrounding cell. ¢..Itismoretoxic in nature than endotoxin, da Itis thermolabile in nature and can be easily destroyed at temperature greater than 60°C, 39 Reference Guide for Foreign Pharmacy Licensing Exam Questions and Answers- 2nd Revised Edition 288. Health Brief Model is used to study patient behavior about: 1. Howtopreventillness. Il. How to stick to a proper diet. TI. — How to take the drugs. Tonly Tand I only Tand Il only I, Hand Ill only eege 289. The chances of noncompliance are high if I. _ Prescribed medication is too costly to buy. Hl. Patientage is over 60 years. IIL. Patients do not know the importance of taking the drugs. a. Tonly b. Land Ilonly cc. Hand Monty d. I, Mand Ill onty 290. The evaluation of a placebo can be done by : 1 Doubleblindstudy Il Single blind study IH, Pseudo blind study a Tonly b. —_TandIfonly cc. MandItfonly 4. [Mand Ilfonly 291. Which of the following is known as the digestive organ of cell ? www.pharmacyexam.com Krisman a. Lysosome b. Endoplasmic reticulum cc. Mitochondria 4. Golgi body 292. Which of the following is normally NOT found in bacteria but present in human cells ? a. Golgi body b. Mitochondria c. —_Lysosome ¢ Endoplasmic reticulum 293. Which of the following is/are true about Lysozyme? I. _Itis generally known as the power house of a cell. II, __Ithelps in the removal of damaged cell. I, Itcontains a bactericidal agent such as lysozyme that kills bacteria before it damages the cell. Tonly Tand Il only Hand Il only I, Hand Ill only aeoe 294, Which of the following is a powerful nonverbal form of communication? Kinetic Proxemic Effective listening Gesture aege 295. Which of the following israte limiting step in synthesis of RBC? a. Availability of pre-erythrocytes, b. Availability of hemoglobin Reference Guide for Foreign Pharmacy Licensing Exam ‘Questions and Answers- 2nd Revised Edition ¢. Oxygen transport capacity d. Presence of erythropoietin 296. Which of the following helps in absorp- tion of vitamin B,,? a. Erythropoietin b. Intrinsic factor c. Hemoglobin 4. Apoferritin 297. Which of the following is a good source of Folic acid ? a Carrots b. Green vegetables ce Milk dish 298. The life span of RBC is a. 90 days b. 30. days c. 120 days d 60 days 299. Whichof the following aboutiron is/are ‘rue? I. __ Reticuloendothelial cells, liver hepa- tocytes and apoferritin are important iron storage sites. I. _Apotransferitinis generally combined with iron in the small intestine, which lateris secreted into the blood. TIL The spleen and bone marrow are im- ortant sites for destruction of RBC. Tonly Tand I only Hand If only 1, Hand IN] only pegs www.pharmacyexam.com Krisman 300. The lack of function of bone marrow generally results in: a, Sicklecell anemia b. —Aplasticanemia ©. Megaloblastic anemia d. Pernicious anemia 301. Polycythemia vera is defined as an: a, Increase of WBC. b. Increase of RBC. €. Increase of lymphocytes. 4. Increase of thrombocytes. 302. Whichofthe following organismsis found in normal flora of the human respiratory tract? a. 8. Pyrogen b.S.Viridan c, Staphylococcus aureus 4. S. epidermis 303. Which of the following organisms is/are responsible for causing meningitis in humans ? LN. meningitis 18 H. influenza UI. S. pneumonia a Tonly b. Land Ionly cand IIfonly d I, Mand If only 304. Whichof the following organisms is re- sponsible for causing 80% or more of bone infections ? a. Spyrogen b. S. aureus c. Cidifficile 4. M. pneumonia 41 Reference Guide for Foreign Pharmacy Licensing Exam ‘Questions and Answers- 2nd Revised Edition 305. Which of the following about lyme disease isNOT TRUE? a Theorigin or occurrence of this dis- ease is the bite of ticks. b. It can be identified by the presence of annular skin lesions found on the thighs and buttocks. c. The bite of a tick is extremely painful. 4. Thedisease is usually associated with vs and neurological complications. 306. Which of the following gram-negative organisms is responsible for causing most uri- nary tract infections ? a. S.aureus bd. E.Colli c. _K-pneumonia 4. Praeruginosa 307. The major biotransformation site of a 401. Liquidity generally expresses a pharmacy’s ability to meet its: a Assets b. Current liability c. Inventory d. Prepaid expenses 402. The acid test generally measures a pharmacy’s : Financial position Liquidity Profitability Inventory 403. Which ofthe following is generallyNOT included in current assets ? a Cash b. Accounts payable cc. Accounts receivable Inventory 404, Which of the following would generally be considered the fixed assets of a pharmacy? 52 Reference Guide for Foreign Pharmacy Licensing Exam Questions and Answers- 2nd Revised Edition a. Inventory b. —Fixturesand equipment | Cah d. Accounts receivable 405. Allofthe following can be considered the current liability of a pharmacy EXCEPT a, Accounts payable. b. Notes payable within I year. c. Accrued expenses. d. Notes payable beyond 1 year. 406. Find out the Acid test (quick ratio) of Manancare Pharmacy from Table 1? a 200/t fia ce tall a oi TABLE 1 A CURRENT ASSETS Cash $50,000 AIC receivable $75,000 Inventory '$ 100,000 Prepaid expenses $ 10,000 Total current assets $235,000 B FIXED ASSETS Fixtures and equipment $30,000 Deposits $5000 Total fixed assets $35,000 TOTAL ASSETS $270,000 C CURRENT LIABILITIES AIC payable $70,000 Notes payable (1 yr) $5,000 Accrued expenses $10,000 Total current liabilities $85,000 www.pharmacyexam.com Krisman D LIABILI Notes payable (>1 yr) $ 20,000 Total liabilities $105,000 Net worth $ 165,000 Cost of goods sold $ 490,000 407. Which of the following does NOT measure the pharmacy’s liquidity ? Acid test ratio Current ratio Net sales to inventory Inventory to its net working capital ae se 408. Total liabilities to net worth ratio of Manancare Pharmacy is : Acceptable Below expectation Exceeds the expectation Cannot be calculated eege 409. The investment in fixed assets of MananCare Pharmacy Exceeds the requirement Is below the requirement Meet’s the requirement Cannot be calculated aege 410, Manancare Pharmacy wants to sell its pre- scription files. The Manancare Pharmacy own- ers asks $350,000 for the existing prescription file. ‘The Manancare Pharmacy provides the follow- ing data upon request. Total new RX dispensed in past years. $ 80,000 The % of Rxthathasone 40% ormore refill left 53 Reference Guide for Foreign Pharmacy Licensing Exam Questions and Answers- 2nd Revised Edition The average RX price $50 Net profit % % 15 What would be youranswer to the owner of the pharmacy? Price is okay. Price is too high. Price is breaking even. Cannot be calculated. aoge 411. “Manancare Pharmacy” marksdown the price of analgesic balm from $3 fo $2. If the mark down of the price increases the sales of analgesic balm from 60 tubes to 80 tubes, what would be the coefficient of elasticity of this prod uct? aod b 2 ce 025 a 05S 412, When relative change in revenue is same as the relative change in price, itis known as: 1. Unitary elasticity I Inelastic demand MI. Elasticdemand a Tonly b. —_TandIonly c. Tand Ifonly 4.1, Mand Ill only 413, Find out the retail price of abox of insulin syringes ifthe cost complement ofthe productis 55% and the cost of one box of insulin is $9.00. a $4.95 b. $16.30 ee Sis ad $13.95 www.pharmacyexam.com Krisman 414, Find out the % markup of Vasotec pre- scription if30 tablets of Vasotec 5 mg retail price is $75 and the cost of the drug is $45 a 55%) be c. 66% da 10% 415. Find out the retail price of one box of insulin syringes if Thecostofcomplement = 55% The known retail markup = 45% The cost of syringes =$9.00 a 495 b. 13.95 © 1630 4 15.1 416. For Manancare Pharmacy, the total rent for the whole store including the Phar- macy department is $10,000. The size of the phar- macy is 600 fé and the size of the whole store is 5000 fe? . On the basis of above figures, what ‘would be the rent of the pharmacy alone? $1000 $2000 $1200 $800 aes 417. The funding for Medicare programs is generally obtained from: TL. —_Social security taxes TL _ Premiums paid by participant TI. State government a Tonly b. Land Ionly ¢. Tand Ill only 4. LM and Ifonly Reference Guide for Foreign Pharmacy Licensing Exam Questions and Answers- 2nd Revised Edition 418. Ina patient cost sharing plan, when a patient has to pay a specified amount of the cost of prescriptions and a third party wil pay the remainding cost of prescriptions, it is known as: Copayment Coinsurance Deductible Retrospective payment pegs 419. A person who works for an insurance company, provides the statistical data that in- dicates the risk associated with serving the popu- lation, and determines the premiums to cover all the estimated expenses is known as: a. Pharmacy manager b. Actuary c. Sponsor Vendor 420. ‘The maximum amount that will be paid by a third party to a pharmacy when the drug isavailable from more than one source is defined as ‘Maximum allowable cost (MAC) Estimated acquisition cost (EAC) Actual acquisition cost (AAC) Average wholesale price (AWC) aege 421. When a patient pays a full predeter- mined amount to the provider at the begin- ning of each month it is known as: a. Concurrent reimbursement b. Prospective reimbursement c. Retrospective reimbursement 4. Cashreimbursement 422. The increase in the number of taking the foreign pharmacy exam isas follows: www.pharmacyexam.com Krisman Year = Students taking the exam. 1981 350 1982 420 1983 530 1984 600 1985 620 1986 635 1987 700 1988 850 Find out the mean of the above data: a 601 db 588 ©. 720 4. 520 423. What would be the median of the above example? 350 850 610 635 aes 424. A random sample of the blood glucose ‘concentration of 100 patients has a mean of 130 and a median of 155. The frequency dis- tribution of the sample a. Normally distributed b. Positively skewed c. Negatively skewed 4. Cannatbe calculated 425. All of the following can be a shape of frequency of distribution EXCEPT: a. Bell shaped distribution b. —_Skewed shape distribution cc. Ushape distribution d. TT shape distribution 55 Reference Guide for Foreign Pharmacy Licensing Exam Questions and Answers- 2nd Revised Edition 26. What would be the Pearsonian coefficient of skewness if.a sample has a mean of 55 anda median of 45. The standard deviation of the sample is 35. a 0.90 Sy e085 d 035 427. Which of the following about a Bino- mial experiment is NOT true? a. Each trial results in an outcome that is classified as success or failure. b. _Therepeated trials are dependent upon previous experiment. c. The experiment generally consists of n-repeated trials. d. The probability of success remains ‘constant from trial to trial. 128. Whatis the mean binomial distribution if the probability of success is 0.60 in 50 trials ? a5 B38 . 8 a4 429. If the blood pressure measurement of 5 people is 110, 135, 140, 125 and 115. re- spectively . What would be the range of the set of the above observations? a «0 b 30 e125 a 140 430. When pictting t distribution curves, if sample size of 20s taken from a normal popula- tion, what Would be the degree of freedom in the tdistribution? www:pharmacyexam.com 19 10 eege 1. Findout the degree of freedom ina 2x3 contingency table Chi-square test when it is applied to test the hypothesis of independence of two variables? pegs Hane 432. The average length of time it takes stu- dents to finish an exam is 180 minutes, with a standard deviation of 36 minutes. A new ex- amination procedure using modern comput- ers is being tested. A random sample of 50 students had an average examination time of 150 minutes, with a standard deviation of 40 minutes under the new system. Test the hypothesis that the population mean is now less than 180 minutes. This hypothesis would resultin ? a. Onesided b. Twosided c Three sided da. ‘Cannot be calculated 433. In protein, Amino acids are joined co- valently by : a, Hydrogen bond b. Peptide bond c. Oxygen bond 4. Disulfidebond 434. The secondary structure of protein consists of : 1 Alfa-helix UL. —_ Beta-sheet MI —_Beta-bend Reference Guide for Foreign Pharmacy Licensing Exam Questions and Answers- 2nd Revised Edition Tonly Tand Il only Tand IIT only 1,1, and I only pose 435. The denaturation of protein can occur in the presence of : lL Heat I Strong acid TI. Organic solvent a. Tonly b. Tand only ce. and {ff only d I, Mand If only 436. Which of the following about sickle cell anemiaisNOT true? a. __Itisa genetic disorder resulting from the production of a variant hemoglo- bin, b. _Itischaracterized with pain, lifelong hemolytic anemia and tissue hypoxia. ¢. Thereplacement of leucine atthe sixth position of the Beta-globulin chain for glutamate is generally responsible for causingit. d. The form Hbshas an extremely low solubility compared to Hba which results into the aggregation of molecules that form or create sickle shaped red blood cells. 437. The enzyme with its cofactor is generally known as: ‘Coenzyme Holoenzyme ‘Apoenzyme Prosthetic group aege www.pharmacyexam.com Krisman 438. The process in which the release of en- ergy from energy rich molecules such as glucose and fatty acid oceurs in mitochondria is known = a. Oxidative decarboxylation b. Oxidative phosphorylation c. Oxidative deamination 4. Oxidative dehydrogenation 439. The breakdown of complex molecules such 4s protein. lipid and polysaccharide into simple molecules such as carbon dioxide, water and ammonia is known as: a. Aerobic glycolysis, b. —_Catabolie reaction ©. Anabolicreaction 4. Gluconeogenesis 440. What would be the end productof gly- colysis in the cell with mitochondria ? a Glucose b. Glycogen ©. Pyruvate d. Lactate 441. Which of the following substance lev- els is found to be deficient in a patient with GOPD deficiency? a, Alfa-antitrypsin in reduced form b. —_Bradykinin in reduced form ¢. Glutathione in reduced form 4. Trypsininelevated form 442. Which of the following causes hemolytic anemia in patients with G6PD deficiency? IL. Oxidantdrug IL __ Ingestion of fava beans TI —_Certaintypes of infections 57 Reference Guide for Foreign Pharmacy Licensing Exam Questions and Answers- 2nd Revised Edition a Tonly b. Land I only ©. Wand IIT only 4.1, Mand IIfonly 443. Which of the following should be classi- fied as a disaccharide? a Ribose b. Lactose €. Glycoprotein 4. Glycosaminoglycans 444. The pairs of structure that are mirror images of each other are known as: a Isomers b. —_Enantiomers c. Epimers 4. Muta rotation 445. In humans, the principle storage of glycogen is found in the : u Skeletal muscle IL Liver 1. Spleen a Tonly b.— Tanddfonly cc. Hand Ilfonly d I, Hand Il only 446. Which of the following substances should be classified asa polysaccharide? a Glucose b. Hyaluronic acid c. Sucrose d. Glycoprotein 447. Which of the following agents acts as an emulsifying agent for metabolism of lipid induodenum? www.pharmacyexam.com a. Gastric lipase b. Bile salt c. Pancreatic juice Secretion 448, Whichof the following about steatorrhea isNOT'TRUE? a. Tteauses.afoss of lipid, essential fatty acid and lipid soluble vitamin in fe- ces. b. The oversecretion of bile salt may im- pair the absorption of fat soluble vita- mins. ¢. The inhibition of secretion of pancre- atic juice from the pancreas generally results in steatorthea. 4. Theabsorption of vitamins suchas vitamin C, thiamine, and riboflavin are not affected by the condition of steatorrhea. 449. Which of the following is a building block of membrane of nerve tissue? a, Prostaglandin b. —Spingomyetin c. Thromboxane 4. Leukotriene 450. Thetermcholelthiasisis generally refered toas: a, Obstruction of the stomach by choles- terol stone. b. Obstruction of the pancreas by choles- terol stone. cc, Obstruction of the gall bladder by cho- lesterol stone. 58 Reference Guide for Foreign Pharmacy ‘Questions and Answers- 2nd Revised Edition 4. Obstruction of the spleen by cholesterol stone. 451. Which of the following isNOT a func- tion of Luteinizing hormone? a Itinitiates the testosterone synthesis in the Leydig cells ofthe testis. b. It simulates the process of spermatogenesis. c. It induces ovulation in females. 4d. Itstimulates synthesis of progesterone and estrogen in the corpas luteum, 452. Thetotalenergy required by an individual can be found by calculating : 1. BMR ‘Thermic effect of food MI. Physical activity a Tonly b. Land I only ¢. Wand II only 4. I,[andIlonly (453. Whichof the following is considered pro- tein deficient malnutrition? 1. —_Kwashiorkor TL Marasmus I —Steatorrhea a Tonly b. — Tand only c. Mand Ilfonly 41, IMand Il only 454, Which of the following is NOT a water soluble vitamin? www.pharmacyexam.com icensing Exam Krisman a Thiamine b. Vitamin c. Niacin d. Pyridoxine 455. Which of the followingis a good source of Vitamin K? Cabbage and cauliflower Fatty fish and liver Vegetable oils Yellow and green vegetables and fruit aege 456. ‘Theend product of purine catabolism is: Alantoin Uric acid Hypoxanthine Xanthine aoege 457. Thesmall andcircularextrachromosomal DNA molecules that carry genetic information for future generations in bacteria are known as : a, Lysozymes b. Plasmids Mitochondria d.—Cytoplasm 458. Which of the following RNA types comprises 80% of the RNA in the cell? IL. Ribosomal RNA. Il Transfer RNA II. Messenger RNA only Land II only Mand Itt only 1, Hand III only aoe 459. Which of the following types of RNA carries genetic information from DNA to cy- tosol for protein synthesis ? 59 Reference Guide for Foreign Pharmacy Licensing Exam ‘Questions and Answers- 2nd Revised Edition 1 mRNA 1 tRNA Ml rRNA a. Tonly b. Land I only c. Hand Ilfonly 4. L,Iand Ilonly 460. Which of the following codons are generally known as stop or nonsense codons? oe UAG TL. UGA Tl. UAA a Tonly b. — TandIfonly c. Tand Hf only da J, Mand Ill only 461. Which of the following types of micros- copy is used to observe the unstained living or difficultto stain organisms? a Dark field b. Bright field ¢. Transmission electron d. Scanning electron 462. ‘The mycobacterium species of bacteria is generally tained with: Gram stain Schaeffer-fulton stain Acid fast stain Flagellarstain pose 463. Which of the following classes of cells have no nucleus ? a. Bukaryotic b. —Prokary‘ c. Homokaryotic 4. Heterokaryotic www pharmacyexam.com 464. Bacteria with two or more flagella is ‘generally known as Monotrichous Amphitrichous Lophotrichous Peritrichous ees 465. The moving of bacteria away from the light is known as a. Chemotaxis }. —Phototaxis cil d.—_Escapetaxis 466. To find out if a substance is carcino- genic which of the following tests should be preformed? L Ames test U. —Pyrogen test I. Biopsy of cells Tonly Land I only Mand Ill only 1, Hand III only pose 467. ‘The synthesis of protein and lipids in cells is generally carried out by : a. Mitochondria b. Endoplasmic reticulum c. Golgi apparatus 4. Lysozymes 468. The growth of bacteria remains constant in which ofthe following phases? a. Lag phase b. Log phase cc. Stationary phase d. Decline phase Reference Guide for Foreign Pharmacy Licensing Exam Questions and Answers- 2nd Revised Edition 469. Whichof the following should be consid- cred a physical factor for growth of bacteria ? Vitamins Moisture ‘Trace elements Carbon sources pose 470. When the transfer of genetic informa- tion from one cell to another cell is carried cut by the plasmid, itis known as: a. Transformation b. Transduction ©. Conjugation 4. Transaction 471. Which of the following components of bacteriophage carries the genetic information? L Genome I. Plate and tail fibers Ik, — Tail sheath Tonly Land Ionly Tand Ill only 1, Iand II only pegs 472. The reduction of the numbers of patho- ‘genic microorganisms, up to such an extent they are not able to produce any kind of disease, known as: Sterilization Sanitization Disinfection Gemnicidation ae ge 473. Which of the following is/are used as antimicrobial agents? |. Isopropylalcohol I. Phenol I. Hydrogen peroxide www.pharmacyexam.com Krisman a Tonly b. — TandIfonly c.— Iand Ill only 4. I.Mand If only 474. The process of weakening the disease producing capacity of pathogens is known as: a. Vinulence b. Attenuation c. Pathogenicity dd. Pseudopathogenicity 475. Which of the following organisms is generally responsible for causing traveler's jiarthea ? a. Cl. tetani b. E.coli cS. aureus dV. cholera 476. The presence of bacteria without multi- plication in blood is known as: a. Septicemia = Bacteremia c. Viremia d. ‘Toxemia 477. Measleisclassified as a(n): Airborne disease ‘Waterborne disease ‘Vector transmitted disease Direct contact disease eoge 478. The life spanof RBCis: a 120days b. — Onehourto one day c. 510 10 days d. 30days 61 Reference Guide for Foreign Pharmacy Licensing Exam Questions and Answers- 2nd Revised Edition 479. Which of the following cells generally guards the skin and mucous membranes against infections? I. Neutrophils I. Basophils III Eosinophils a Tonly b. — Tand Ionly ¢. TlandIMonly 4. I, Iand Mf only 480. Which of the following cells are a source of production of Alfa-interferon? a. Fibroblasts b. Leukocytes c. Tlymphocytes 4. Natural killer 481. Whichof the following immunoglobulins provides antibody protection for the fetus? a IM b. IgG c IgA da IgE 482. Whichof the following immunoglobulins are found to be elevated during asthma and other allergic diseases ? a IgM be © IgA a gE 483. The ability of the immune system to identify and respond immediately to foreign substances to which it was exposed previously is known as: www.pharmacyexam.com Krisman a. Specificity b. Memory c. _ Heterogenicity d. — Homogenicity 484, The production of cells in the labora- tory from a clone of cultured cells that make one specific antibody to a specific epitope is known as: a. Polyclonal antibody b. Monoclonal antibody c. _Interleukine 4. Interferon 485. Which of the following vaccines is pre- pared from atoxin? a. Influenza b. Diphtheria cc. Mumps da Tuberculosis 486, Inhumoral immunity, when an antigen rec- ognized by memory cells enters the blood it is knownasa: a. Primary response b. Secondary response c. Tertiary response 4. Zero response A887. The introduction of ready-made antibod- ies into a normal person in order to induce immu- nization against a particular disease is known as: I. Antitoxin I Hyperimmune sera II. Immuneserum globulin a. Tonly b. Land Ionly c. Wand Ilfonly d.— ,MandIITonly 62 Reference Guide for Foreign Pharmacy Licensing Exam Krisman Questions and Answers- 2nd Revised Edition 2493. The disturbance of copper metabolism 488. Allof the following can be classified as anactive immunizationEXCEPT : a. Mumps b.— Antitoxin c. Rabies a. Plague 489. Hemolytic disease of anewbominan Rh negative mother withan Rh-positivefetusis known as: ‘Type I hypersensitivity ‘Type II hypersensitivity ‘Type III hypersensitivity ‘Type IV hypersensitivity Bese 490. Which of the following immunoglobu- lins is a principal mediator of type I hyper- sensitivity ? oo. -tey hh yee a IgG b IgM cE a Teells 491. Which ofthe following isNOT an organ specific autoimmune disorder ? Addisor M. gravis Ulcerative colitis, Goodpasture’s syndrome s disease ee se 492. Which of the following isa characteristic that identifies SLE ? a Joint inflammation b. Butterfly shaped rash ©. Muscle weakness and fatigue d. Severe headache with nausea and vomiting www.pharmacyexam.com may cause Cohn’s syndrome Wilson’s disease Barr syndrome Cushing's syndrome ees 494. Whichof the following osmotic diuretics is useful for treatment of cerebral edema? a Thiazide diuretics b. Mannitol ¢. Spironolactone da Acetazolamide 495, Which ofthe following diuretics reduces the absorption of basic drugs? I. Ammonium chloride I. Ammonium nitrate I. — Calcium chloride a Tonly b. Land I only c. HandIlonly dL, Mand Iltonly 496. Which of the following diuretics has an. aldosterone antagonist property? a. Hydrochlorothiazide b. Spironolactone c. Furosemide d. Bumetanide 497. Which of the following antacids absorbs into systematic circulation and can produce sys- temic alkalosis? a. Aluminum hydroxide b. Sodium bicarbonate Calcium carbonate d. Magnesium oxide 63 Reference Guide for Foreign Pharmacy Licensing Exam ‘Questions and Answers- 2nd Revised Edition 498. Which ofthe following cells take an active part in secretion of acid in the stomach? Epithelium cells Parietal cells Cells of langerhans Epidermal cells aege 499. Which of the following receptors are generally involved in the production of HCl in the stomach? I. Muscarinic receptors UL Histamine receptors TI — Gastrin receptors a. Jonly b. and only ce, Hand I only & [Mand If only 500. Which of the following factors stimu- lates the release of gastrin from antral G cells? I, Distension of stomach by food TI, Alkaline pH of stomach Ml, Digested proteins Tonly Tand IT only Tland II only |. i, Mand Hlonly ao ge 501. Which of the following prostaglandins takes an active part in the production of mu- cosa in the stomach? Prostaglandin G Prostaglandin E, Prostaglandin F Prostaglandin K pege 502. The stoppage or impaired secretion of hydrochloric acid in the stomach is known as: www.pharmacyexam.com Krisman a, Dysphoria b. — Achlorhydria c. — Epistaxis Dyspepsia $03. Which of the following compounds have an antitussive property? 1. Codeine Wi. Dextromethorphan IIL. Benzonatate Tonly Tand If only Mand III only 1, Wand UT only eege 504. Overdose of heparin can be prevented by administration of : a Vitamin K, b. —Protamine sulfate c EDTA 4. Phenobarbital 505. Which of the following drugs inhibits DNA-dependent RNA polymerase enzymes, and thus the formation of m-RNA? 1. Actinomycin I, Mitomycin MM. — Rifampicin a, Tonly b. Land I only c. Hand Ilfonly 4. Land Tfonly 506. Which of the following drugs acts on rep- lication levels of protein synthesis? Streptomycin b. Tetracycline c._ Bleomycin 4. ‘Erythromycin: ) Reference Guide for Foreign Pharmacy Licensing Exam, Questions and Answers- 2nd Revised Edition 7. Which of the following are mechanisms ‘of resistance against chemotherapy agents caused by bacteria? 1 Mutation . Transformation HI. Conjugation a Tonly b. Land Ionly ¢ — Tand Hfonly 4. Mand If only 508. Sulfonamide is found to be ineffective in patients with Blood disorders Pus formation Hypoalbuminemia Slow acetylators The bacteriostatic action of rimoxazole is attributed to inhibition of : eg pose Diphydrofolate reductase Dihydropteroate synthetase DNA gyrase Tonly Land II only Mand Ill only 1, Hand III only aesPp por 510. Which of the following compounds gen- erated by Methenamine acts as a urinary antiseptic? a. Hydrogen Peroxide b. Formaldehyde c. Benzaidehyde a Alcohol 511. Which of the following chemotherapy agents produces its antibacterial action by in- hibiting bacterial DNA synthesis by blocking DNA. gyrase? www pharmacyexam.com Krisman a, Erythromycin b. Tetracycline ©. Ciprofloxacin d. Chloramphenicol 512. Which of the following antibacterial ‘agents is indicated for the treatment of Methicillin resistant infections ? Tetracycline ‘Vancomycin Metronidazole Norfloxacin aege 513. Which of the following penicillins has an acid resistant property ? a. Methicillin b. Penicillin V cc. Oxacillin d— Ticarcillin 514. The common adverse effect associated with the quinolone group of antibiotics is Hepatic dysfunction CNS stimulation Muscle pain Ototoxicity erge 515. Which of the following antibiotics is indicated for treatment of UTI? 1. Ciprofloxacin I. Cotrimoxazole TI —_Nitrofurantoin a Tonly b. Land I only ec. and only 4. I, Hand Ill only 516. Which of the following cephalosporins ‘would be the drug of choice for treatment of men- ingitis? Reference Guide for Foreign Pharmacy Licensing Exam Krisman ‘Questions and Answers- 2nd Revised Edition Boo Cefaclor Cefiriaxone Cefazolin Cephalexin 517. Structure activity relationships would eliminate which of the following side effects of Ampicillin ? Begs Anaphylactic Shock Diarrhea Hepatic necrosis Electrolyte imbalance 518. Whichof the following areadverse effects of aminoglycosides? 1 Ototoxicity I, Nephrotoxicity IM. — Curaremimetic effect eoge Tonly Tand Il only Mand Ill only I, Hand Ill only 519, Which of the following is a principal side effect of tetracycline that restricts its use in children, and pregnant woman after the second trimester? a. Gray baby syndrome b. _Anaphylactic shock c. Defective teeth and bones d. Bone marrow suppression 520. Which ofthe following metabolite prod- ucts of Streptomycin is generally responsible for causing deafness? a, Epianhydrotetracycline b. —_Dihydrostreptomycin ¢. Acrolein d. N-acetyl p-benzoquinone www.pharmacyexam.com 521. Which of the following is a principal adverse effect of Pyrazinamide? Renal failure Red-green color blindness Arthralgia An orange color in saliva, tears, and sputum Bese 522. Which of the following antibiotics in- hibit the synthesis of folic acid from PABA? Rifampicin Clofazimine Dapsone Isoniazid ee ge 523. Ifthe concentration of reactant Alfa is triple in the reaction that is in second order, the rate of reaction would generally increase: 2times 4 times 9 times 8 times enrge 524. Which of the following pyramidine ana- logs is useful in the treatment of cancer chemotherapy? Methotrexate Mercaptopurine Fluorouracil Mitomycin pose 525. Which of the following tests is used to find out the potency of disinfectants? 1. Rideal-walker test I. Chick-martin test I. Moniliasis a. Tonly b. — TandIlonly Reference Guide for Foreign Pharmacy Licensing Exam Krisman Questions and Answers- 2nd Revised Edi Mand only 528. The absence of the secretion of urine by dL, fland If only the kidney is known as: 526. Which of the following is true about a Alopecia alcohol as a disinfectant? b. Anorexia e Anuria L Primary alcohol has less bactericidal d.— Achlorhydria activity compared to secondary and tertiary alcohol. \_-529. Inflammation of the lips is a frequently reported side effect of WH. At70% concentration, alcohol ionizes the best and provides optimum disin- a. Bleomycin fection. b. Isotretinoin cc. Clindamycin II At 70% concentration, alcohol wets 4 Hydroxyurea the skin smoothly and evaporates slowly. 530. The copper ion Cuis representanted as © Cu, Find out the number of neutrons in the a lonly above ion. b. Land Honly c. HandTonly a 2 4 (Mand only b 34 . 2 527. The Rideal walker coefficient of disinfec- i 63. tant can be calculated by : 531. How many iron atoms are present in a Dilution ofdisinfectant-killed ferrous sulfate? [number of moles of iron = 4] Dilution of formaldehyde-killed a. 6.02x 108 microbes in 5 to 7.5 minutes b. 2.4 x 10" c 35x104 b. Dilution of disinfectant-killed a 42x10" microbes in Sto7.S minutes Dilution of phenol -killed 532. How many molecules of C,H,,0, are microbes in 5 to 7.5 minutes required to balance the equation below? c Dilution of disinfectant-killed CH,,0, + 150, = 12CO, + H,O oe “0, + 150, , + Hy Dilution of alcohol-killed 7 microbes in 5 to 7.5 minutes b. 4 c. 6 4. Dilution of disinfectant-Killed a 8 Dilution of #O™-killes microbes in 5 to7.5 minutes www.pharmacyexam.com 6 Reference Guide for Foreign Pharmacy Licensing Exam Krisman Questions and Answers- 2nd Revised Edition 533. Find out the weight of K,CrO, to pre- pare 250 ce of 0.5 M K,CrO, solution? [MW of K,CrO, = 194.2 gm/mole] a 12.20 gm b. 24.27 gm c. 97.1 gm 4, 48.56 gm 534, 3L of N, gasat5.5 atm pressure expands toa final volume of 16L. Find out the final gas pressure of N, in mmHg. [ Assume that temp is constant] a. 5.3 atm b. 1.45 atm ce 29 atm 4. 12.5 atm 538, Calculate the partial pressure of H, if the total pressure of the mixture of gases is 10atm and the partial pressure of He, is 4.6 atm. a :10.0atm b. S.4atm c. 46am 4d 23atm a 1.03 539. If 4.4x 10* molecules of H, gas effuses b. 183.75, V~ through offices in 200 seconds, how much N,(g) 365.10 ‘would effuse through the same offices in 200 sec- a 135.20 onds? 535. AcylindercontainsN, gas at2.atm ata temperature of 30°C. The volume of gas found was 201 at this temperature. Find out the vol- ume of gas ina liter at 50°C. [Assume the pressure remains constant] a 10L. b 21.3L ce 25.6L da 45.2b 536, Find out the volume of Cl, (gas) occu- pied at a temperature of 30°C and a pressure of 730mm Hg. The weight of Cl, (gas) is S0gm. {molecafar weight=70.9 1 gnvmole] [R= 0.082 Lx atm/mole deg] 10.5L 12.4L 18.24L, 25.5L ap ge 537. Find out the pressure of O, at 125°C if the volume of O, remains the same. {Assume the gas behaves ideally at O°C and at 1 atm pressure.] ‘www.pharmacyexam.com ae 210s b. 32x 10+ c, 1.176 x 104 d 88x 104 540, How many angstroms are equal to | na- nometer? 0.1 Ae 1.0 Ae 10 Ae 100 Ae Boge (SAL. Find out the wavelength of light from a mercury vapor lamp witha frequency of 10x 10!"HZ. The rate of light in a vacuum is 2.998 x 10° mis. 530 nm 299 nm 135 am 480 nm Bese 542. The molecular geometry of aCH, (meth- ane) molecule is: Reference Guide for Foreign Pharmacy Licensing Exam Questions and Answers- 2nd Revised Edition a Lineas b. Angular ce. Tetrahedral d.—Trigonal bipyramidal $43. ‘The transfer of molecules from a solid phase directly to a vapor phase is known as: a Melting b. Sublimation c. Freezing Evaporation 544, Find out thet,, of H,O,,, if decomposi- tion constant in first order kinetic is 0.693 x 10 second", a: 10 seconds b. 100 seconds c. 200 seconds d— Lhour P45. ‘The decomposition of ethylacetate in the presence of (excess) water follows the : CH,COOCH,+ H,0 = CH,COOH + C,H,OH a Zeroorder b. First order c. Pseudo first order d.— 2ndorder 546, {the concentration of reactantAlfaishalf in the reaction thats second order in kinetic, the ‘ate of reaction is multiplied by factor: a 8 b 14 = - a 12 547. ‘The decomposition of Calcium carbon- ate inthe presence of heat is as follows: www.pharmacyexam.com A = caca3 > C20, + CO, The addition of a 5 liter solution of Calcium oxide in the above mixture would change the re- action in which direction ? a. Direction b. Direction B c. Remains same d. Cannot be predicted 548, Which of the following is the correct the ‘Way of expressing strength of the following acids in descending order ? HCO, >HCIO, >CH,OH >NH,’ NH, >HCIO, > H,CO, > CH,OH HCIO, > H,CO, > NH, > CH,OH CH,OH > NH," > HCIO, > H,CO, ao ee 549. Find out the Ksp of Li,PO, (0.5 gm/ Y 100 ce) solution. a 5x10? b. 9.22 10" ce 4x10" d 25x 10° 550. Thechlorination of Nitrobenzene gener- ally results in Ortho Chloronitrobenzene Para Chloronitrobenzene Meta Chloronitrobenzene Chlorabenzene peo 551, The nitration of benzaldehyde will result Meta isomer Ortho isomer Paraisomer Ortho and para isomers aege 69 Reference Guide for Foreign Pharmacy Licensing Exam ‘Questions and Answers- 2nd Revised Edition 552. 2 Methyl 2 Propranolol is classified as a: a. Primary alcohol b. Secondary alcohol c. Tertiary alcohol d._ Polyhydroxy alcohol 553, Whatisthe IUPAC name of the follow- ing compound ? are ° Benzylphenone Acetyl phenyl ketone Acetyl benzyl Propranone aege 554. The oxidation of secondary alcohol generally results in: a. Aldehyde b. Acid c. Ketone d. Primary alcohol 555. The IUPAC name of the following compoundis: cH, -CH, COOH, a Beta chlorobutyric acid Alfa- chlorobutanoie acid Alfa- chloropropanoic acid 2 chloropropanoic acid eege 556. Which of the following compounds has the lowest PKa value? a. Methylamine >. Ammonia ce Aniline 4. Methylethytamine www.pharmacyexam.com Krisman ,-557. Whichof the following compounds pro- duces a high concentration of H,0* ions when dissolved in H,O? a. Penitrophenol b. 2,4,6 trinitrophenol c Phenol 4. Ortho nitro phenol {_-558.__The risk of teratogenesis isthe high- est during which trimester ? a Firsttrimester b. Second trimester ¢. Third trimester Fourth trimester 559. Which of the following categories has shown the highest risk to the fetus? a. Category A b. Category B c. Category C d. Category X 560. Which of the following antibiotics is the drug of choice for treatment of UTI in pregnant ‘women in their first trimester? a. Tetracycline b. SMZ/TMP ciprofloxacin 4. Metronidazole 561. Allofthe following drugs are teratogenic in nature and should be avoided by pregnant women EXCEPT : a. Finasteride b. —Triretinoine cc. — Benztropine 4 Lithium carbonate 0 Reference Guide for Foreign Pharmacy Licensing Exam Questions and Answers- 2nd Revised Edition 562. Which of the following are essential fatty acidsin humans? L Linoleicacid I, Linolenic acid Il. Arachidonic acid Tonly Tand Honly Tand Il only I, Mand IM only aoge 563. Ifapatient with 30% of hematocrit lost 1000 cc of blood in a car accident, what would be the dose of iron dextran (50 mg/ml) to re- place the iron loss ? a 3m b& 6 ml cil a 2 al 564. The principal energy-carrying molecule in acellsis: a AMP b. ADP c. ATP 4. NADP 565. Whichofthe following drugscan be used as an adjunctive therapy with Coumadin in a patient with cardiac valve replacement? Lidocaine Dipyridamole Phenytoin Diazepam — percentint. Bose 566. Which ofthe following parenteral routes should be avoided with heparin? a S.C bo LM. www.pharmacyexam.com Krisman «. WM d.—_Inermittent I.V. 567. Whichof the following tests is widely used to monitor heparin therapy ? a PIT b APTT cc. Whole blood clotting time a INR 568. All ofthe following would increase the risk of bleeding with Coumadin therapy EXCEPT: a. Cefamandole b. Cimetidine c. Phenobarbital 4. Chlorpropamide 569. Which of the following drugsis used for treatment of hemophilia? a. Aminocaproic acid b. Hemin c. Antihemophilic factor 4. Estrogen 570. Which of the following mechanical contraceptives has shown the least risk of preg- nancy? a. Diaphragm b. Vaginal sponge Condoms a UD 571. Rank the following glucocorticoids according to their expected half lives: a. Cortisone > Triameinolone > Betamethasone > Hydrocortisone b. Betamethasone >Triamcinolone> Hydrocortisone > Cortisone n Reference Guide for Foreign Pharmacy Licensing Exam ‘Questions and Answers- 2nd Revised Edition ¢. Triameinolone > Hydrocortisone > Cortisone > Betamethasone 4. Hydrocortisone > Betamethasone > Cortisone > Triameinolone 572. Which of the following corticosteroids has the highestmineralocorticoid activity ? a. Prednisone bv. — Methylprednisolone ce. Dexamethasone 4. Fludrocortisone 573, Aminoglutethimideis indicated for the treatmentof: a. Bleeding disorder b. Cushing's syndrome cc. Hemophilia d. Prostatic cancer 574, Rank the following insulins in de- scending order according to their duration of action: a. Prompt Insulin Zn > Protamine Zn > Tsophane Insulin Zn > Insulin lispro b. _Protamine Zn Susp> Isophane insulin Protamine Zn insulin > Insulin lispro ¢. Insulin lispro > Protamine Zn > Prompt Zn > Isophane insulin 4. Protamine Zn> Prompt Zn> Insulin lispro> Isophane 575. Which of the following sources of insu- linsis the least antigenic in nature ? a. Sheep b. Beef c. Human d. Pork www.pharmacyexam.com Krisman 576. “Insulin resistant” is defined as when a patient needs more than how many units of insulin per day? ic 80, ibe 120 c. iso dG 200 577. Which of the following symptoms helps to differentiate diabetic ketoacidosis from hypoglycemia? I. Rapid pulse. II. The presence of glucose and acetone in urine. I, Acetone breath. a. loaly b. Land Nonly c, Iand ITonly aI, Mand If only 578. Thyroid 1/4 grains is equivalentto. a 16 mg b. 32 mg c 65mg 4. 8 mg 579. What would be the volume of 5% potas- sium iodine that is required to mix with a 10% solution of potassium iodide to make a7.5%, 500 cc solution of potassium iodide ? a 250 ce b. 500 ce ce 150 ce d 200 cc 580. Which of the following diuretics acts by inhibition of Na’K*ATP-ase activity? nR Reference Guide for Foreign Pharmacy Licensing Exam Questions and Answers- 2nd Revised Edition a, Tiamterene b. Spironolactone ©. Amiloride d. Furosemide 581, How much Mannitol is present in 150 cc of 15% solution of Mannitol ? a 15.0 gm b. 22.5 gm ce. 44.0 gm da 75 gm 582. If 75 ml of 25% Mannitol solution is diluted up to 800 cc with SWFI, what would be the percentage of the final concentration of Mannitol ? a 34% 6. 36 % « $6 % d 234 % 583. Lanoxin (digoxin) can be indicated for the treatment of all of the following EXCEPT Atrial flutter Atrial filtration Ventricular fibrillation CHF pers 584, If apatientis taking Lanoxin (digoxin) for treatment of CHF and is then diagnosed with myxedema, the dose of the drug should be: Kept the same Reduced and monitored Increased and monitored Omitted eee 585. Which of the following is true about the toxicity associated with Digoxin ? wwwpharmacyexam.com Krisman 1. Avoid the use of potassium supple- ‘ments in patients with complete heart block because of Digoxin. Mi, Severe sinus bradycardia can be treated with atropine I. Forcorrection of athythmia, lidocaine or phenytoin can be used. a Tonly b. Land only c. Hand Ilonly d. I, Hand Il only 586. Which of the following adverse effects is commonly reported with the prolonged use of ‘Amrinone? a SLE b. Thrombocytopenia cc. Agranulocytosis d. Peripheral neuropathy 587. Rank the following Nitroglycerine prod- ucts according to their onset of action: a. Nitrostat SL> Nitrolingual spray >Nitrostat IV > Nitrodur TDS b. —_Nitrostat IV > Nitrostat SL > Nitrolingual spray > Nitrodur TDS . _ Nitrolingual spray > Nitrostat 1V > Nitrostat SL > Nitro TDS d.—_ Nitrostat IV >Nitrolingual spray > Nitrostat SL > Nitrodur TDS 588. Which of the following is the correct way of administering Nitroglycerine at the firstsign of, acute anginal attack? B Reference Guide for Foreign Pharmacy Licensing Exam Questions and Answers- 2nd Revised Edition Dissolve two tablets under tongue and repeat steps every 30 mins until pain iscontrolled. b. Dissolve one tabletunder the tongue at the onset of attack and repeat every 4 hours for 2 days. c. _Dissolve one tablet undertongue at acute onset of attack, repeat every 5 minutes until complete reliefs achieved, Do not exceed more than 3 tablets withina 15 minute interval 4, Dissolve one tablet under tongue at on- set of attack and prepare patient for administration of ILV, Nitoglycerins. 589. The Cardiotoxic effects of quinidine can be reduced by administering : a, Sodium bicarbonate b. —Phenytoin c. Sodium lactate 4. Diazepam 590, Ringing in the ear, headache, nausea, ver- tigo, and vision disturbances are reported side effects of: a. Procainamide b. — Phenytoin cc. Quinidine d.— Flecainide 591. Which of the following salts of Quinidine contains the highest amount of anhydrous quini- dine alkaloid ? a. Quinidine sulfate b. —— Quinidine gluconate ¢. Quinidine polygalacturonate dd. Quinidine hydrochloride www.pharmacyexam.com Krisman 592. The use of Procainamide is contraindi- cated in patients with: Anthythmia Myasthenia gravis Ulcerative colitis, Zollinger-elision syndrome eege 593. Which of the following drugs may pre- cipitate Lupus erythematosus-like syndrome? 1 Procainamide I. Hydralazine Ti. Lidocaine Tonly Land Il only Mand Il only 1,11 and Ill only eege 594. Patients with congestive heart failure have toavoid which of the following? a, Procainamide b. —_Phenytoin c. Disopyramide d. Lidocaine 595. Which of the following are reported side effects of Lidocaine ? 1 Malignant hyperthermia I. —_CNStoxicity MI, Hepatic only Land Honly Mand Il only 1,11 and JIT only eege 596. Which of the following classes of antiar- thythmic agents is restricted only for life- threatening arthythmia ? 74 Krisman Reference Guide for Foreign Pharmacy Lice Questions and Answers- 2nd Revised Edition 1g Exam a Class IA. b. Class IB c.— ClassIC 4. Class IT 597. Which of the following potassium salts contains the highest meq amountof potassium? a. Potassium chloride b. Potassium bicarbonate cc. Potassiumcitrate d. Potassium gluconate 598. Which ofthe following vitamin A deriva- tives helps with visual adaptation in darkness? a Retinol b. —_BetaCarotene c. Retinoic acid d. Retinal 599, Which of the following are precur- sors of vitamin D? I. Ergosterol I 7-dehydrocholesterol MI. —_Dihydrotachysterol a. Tonly b. Land Ionly c. Iand I only da. 1, Iand Ill only 600. Which of the following calcium salts has the lowest amount of elementary calcium? a. Calcium carbonate b. Calcium gluconate c. — Calciumcitrate d. Calciumacetate 601. Urinary alkanizers may increase the excretion of all of the following drugs EX- CEPT: www.pharmacyexam.com Methotrexate a b.— Quinidine c. Salicylate 4. Methenamine 602. Which of the following drugs is indicated for the removal of excess K* ions from blood? a. Calcium phosphate b. EDTA c. Sodium polystyrene sulfonate d. Deferoxamine 603. Which of the following drugs is indicated asan appetite stimulant agent? a. Epotein b. — Megestrol acetate c. Pyridoxine 4. Ensure 604. Which of the following drugsis indicated for the treatment of Benign prostatic hyperplasi a. Danazole b. Finasteride c. Medroxy progesterone d. Mannitol 605. Which of the following is a principal adverse effect of anabolic steroids? Hypercholesterolemia Peliosis hepatis Amenorthoea Anutia 606. Prostate cancer is known to be sensitive Progesterone Diphydotestosterone Estrogen Follitropine 18 Reference Guide for Foreign Pharmacy Licensing Exar Questions and Answers- Ind Revised Edition 607. Oxytocin generally causes: Dilation of uterine muscles. Dilation of coronary arteries. Contraction of uterine muscles. Contraction of prostate gland. eee 608. Which of the following drugs is/are indicated for treatment of Paget's disease of bone? L Alendronate sodium Il. _Etidronate disodium II. Tiludronate sodium a. Tonly b. Land only Hand Il only 41, Mand Ill only 609, Which of the following insulins has a rapid onset of action and the shortest duration of action? Regularinsutin Prompt insulin Zn Insulin lispro Insulin Zn suspension eege 610. Which of the following Sulfonyl urea agents should be avoided by geratic patients ? ‘Tolbutamide Chlorpropamide Glyburide Glipizide pose 611. Table sugar as source of glucose for the treatment of hypoglycemia should not be used by patients with: a. Metformin b. — Acarbose c.—— Glyburide 4 Insalin www.pharmacyexam.com Krisman 612. Which of the following diabetic agents has the greatest risk of hepatotoxicity? a. Metformin b. Troglitazone c.— Glyburide 4 Insulin 613. Patients with hypersensitivity to thiazide diuretics may need to avoid: I. Diazoxide Il. Metolazone TI. Hydrochlorothiazide a Tonly b. Land only ¢. Handitfonly dT, Mand UT only 614, Which of the following antihyperthy- roid agents can be safely administered to preg- nant women ? a. Methimazole b. —_Propylthiouracil ©. Potassium iodide 4. Levothyroxine 615. Which of the following isa therapeutic use of Desmopressine? Diabetes insipidus Hemophilia Williebrand’s disease Tonly Land Il only Mand Ilonly 1,11 and Ill only Boge Ree 616. Octerotide acetate generally inhibits the secretion of : 16 Reference Guide for Foreign Pharmacy Licensing Exam Questions and Answers- 2nd Revised Edition a. Thyroidhormone b. Growth hormone ce. Noradrenaline d. Serotonin 617. Which ofthe following drugsis indicated for treatment of hyperammonemia ? 1. Lactulose TL Sodium benzoate and sodium phenylacetate TI. — Methamphetamine Tonly Vand I only Tand [if only 1, Iand Ill only aege 618. Protonged treatment with Bromocrip- tine should be monitored through: Retina function Pulmonary function Renal function Hepatic function ae gp 619. Probenecid may elevate the serum con- centration of which of the following drugs? Methotrexate Retrovir Pantothenic acid. Ber Tonly Land If only Mand Mf only L,Iand Il only apge 620. Cyanide poisoning can be treated with: Deferoxamine Mesna Diazepam Amyl nitrite aoge www.pharmacyexam.com Krisman 621. Whichof the following drugs should be used for treatment of Cyclophosphamide induced Heystitis? Dimercaptrol Leucovorin Ca Mesna Vitamin K aere 622, Which of the following drugs antagonizes the action of opioid drugs? Naloxone ‘Nalmefene Naltrexon Reo Tonly Vand IL only Mand Ilfonly I, Hand I only aege 623, Which of the following drugs antago- nizes the toxic effect of benzodiazepine over- dose ? a Deferoxamine b. Fluoxetine c. — Flumazenit Chlorpromazine 624, Which of the following informations is NOT true about activated charcoal ? a. Ttisindicated for treatmentof drug intoxication in an unconscious patient. b. __Itis carbon residue derived from or- ganic material by exposing it to an oxidizing gas compound of steam, oxygen and acid at a high temperature. ©. Activation of charcoal surfacesin- creases the adsorption properties. vas Reference Guide for Foreign Pharmacy Licensing Exam Questions and Answers- 2nd Revised Edition Each gram of charcoal is capable of, bind 100 mg to 1000mg of drug. 625. Alkalinization of urine increases the bioavailability of : a. Propranolol b. Verapamil © Cimetidine d.— Flecainide 626. Whichof the following Beta-blockers has Beta-I receptor selectivity ? a. Pindolol b. Sotalol Propranolol 4. Acebutolol 627. Apositive Comb’s Test helps to identify hemolytic anemia associated with: a. Chloramphenicol b. — Procainamide cc. Methyldopa d.— Flecainide 628. | Which of the following tests isem- ployed to evaluate tablet dosage form ? 1 Weight variation IL Disintegration TH Dissolution a Tonly 6. Land only cc. MandIffonly 4 1, MandITonly 629. Breakdown of tablets into smaller par- ticles or granules is defined as the process of: a. Disintegration b. Dissolution www.pharmacyexam.com, Krisman cc. Rupturization 4. Granulation 630. Which of the following is/are pro- cessing problems for a manufacturer of tablets ? L Capping I. Mottling ML. Picking a. only b. Land Honly Mand Ill only 4. L.MlandIifonly 631. Which of the following ingredients is used as a disintegrant for tablet dosage form ? a. Starch b. Lactose ic Tale d. Stearic acid 632. Which of the following ingredients increases the flow property of granules ? a Acacia b. Comstarch c. hays 4. Mannitol 633. Whichof the following diluents should be avoided with Tetracycline ? a. Lactose b. Mannitol ¢. Calcium phosphate d. Microcrystalline cellulose 634, Forwater sensitive products, which of the following diluents should be used in formulations ? 78 Reference Guide for Foreign Pharmacy Licensing Exam ‘Questions and Answers- 2nd Revised Edition 1 Lactulose W._Diabasic calcium phosphate IL Calcium sulfate Tonly Land Ionly Wand IIT only I, land Ill only aooe 635. The reaction that may be caused by certain amine drugs in the presence of lactose diluent is known as : a. Redox reaction b. _Leonatd reaction c. Millard reaction d Axis reaction 636. Which of the following is an ex- ample of hydrolysed starch ? a Emdex db Cerelose c. Dextrose d— Avicel 637. Which of the following diluents is used for manufacturing chewable tablets ? a Lactose b. Menthol Sorbitol @.—— Microcrystalline 638. Which of the following is/are naturally occurring gums? 1 Acasia IL Tragacanth ML Gelatin a Tonly b. Land I only c. TandIIlfonly dL, Hand Mlonly www.pharmacyexam.com Krisman 639. Which ofthe following ingredients is employed as a disintegrantin the manufacturing of tablets ? Sorbitol Dibasic calcium phosphate AcDi-sol 50% solution of glucose eege 640. Which of the following is true about the coating of tablets ? 1. Itmasks the taste, odor or color of the drug I It controls the release of drug from the tablet. IIL Itprotects the drug from disaster effect of stomach acid. Tonly Tand Il only Tl and II only 1, Hand IIT only peeP 641. Lubricants are intended to : a. Increase the flow property of gran- ules. b. Reduce the friction between the tablet and the walls of die cavity in which the tablet is generally formed. ¢. _ Disintegrate particles of the tablet in the stomach. 4. Improve the color property of drugs. 642. In which of the following drug informa- tion literature are the citations arranged alphabeti- cally by first author and then by subject head- ings? 9 Reference Guide for Foreign Pharmacy Licensing Exam ‘Questions and Answers- 2nd Revised Edition a. Pharmacy Law Digest b. Clinical Pharmacy and Therapeutics a Index Medicus d. PDR 643. Which of the following references is use- ful to find out which drugs have been withdrawn from the U.S. market ? a. AMA Drug Evaluation b. — Drugdex c D-List ad PDR 644, Which of the following reference sources helps for physical identification of tablets and capsules ? T Identidex Il Red Book TI Blue Book a Tonly b. Land Ionly cc. Tand Ill only a all 645, Tofind outthe wholesale and retail price of particular product, one can use which of the following 1 TheAmerican Druggist Blue Book Drug Topics Red Book Ml, DeHaen Drug Data a only b.— TandIonly c.— Iand IT only d. All 646. Which of the folowing HMO models has the highest percentage of HMO membership ? a. Staffmodel HMO_ wwww.pharmacyexam.com Krisman b. Group model HMO c. Network model HMO 4. IPA model HMO. 647. Physicians participating in IPA HMO’s are normally reimbursed by L Capitation I Discounted FFS I. Salary a. Tonly b. Land Ionly c. Hand Il only ad All 648. 1 millimicron is equal to how many angstroms ? 1a 100 A° 10a° 1000 A° Bese 649, Which of the following is TRUE abou a USP standard dropper? I Theexternal diameter of the dropper must be 3 mm atits delivery end. I Whenheld vertically itshould deliver 20 drops of water, the total weight of, which is between 0.9 gand 1.1 g at 25°C. TI Itwas first recommended by the Brussels Conference of 1902 for international adoption. a Tonly b, — TandIlonly Hand Ill only a All Reference Guide for Foreign Pharmacy Licensing Exam Questions and Answers- 2nd Revised Edition 650. Whatis the specific gravity of 2Lof Car- bon tetrachloride having a weight of 2500 gm ? a 225) On ce. 0,008 a 1.25 651. Whichof the following is the correct unit for density ? a gm/ce b. colgm cc. gvmm? dd gmem/mm? 652. What would be the pH of the solution, if the concentration of hydrogen ions in the i$ 0.0001 g-ion/L.? aege Bow 653, Ina statistic, Bias refers to the closeness of measurement. thereproducibility of result. systematic difference from the true value. d. All 654. Vasculitis angitisis defined as: a. —_Asense of suffocating, b. A patchy inflammation of the walls of small blood vessels. cc. _Anexcessive variation in size between individual red blood cells. 4. Failure ofthe kidney to produce urine. 655. Whichof the following abbreviations is used to describe a Podiatrist ? www.pharmacyexam.com Krisman a MD b= DDS ©. DPM id NP 656. How much sodium bicarbonate is re- quired to prepare 300 cc of 0.05 N solution of NaHCO, ? [ molecular weight = 84 gm/mole] a 84gm b. 126m c 42am da 0.84 gm Ss CH,=CH, is classified as: a. SP* hybridization b. — SP*hybridization c. SPhybridization da. P* hybridization 658, Which of the following bonds requires the greatest amount of energy ? a C=0 vb «ACI e C=C do NA 659. If 1 part of solute requires 10 to 30 parts ‘of solvent, it should be defined as: Soluble Freely solubie Slightly soluble Sparingly soluble eoge 660. Calculate the [H30°] concentration of a 2 0.09M solution of ammonium chloride. Assume that Kb for ammonia is 1 x 104 and Kwis 1 x 10 a 102 b. -24x105 81 Reference Guide for Foreign Pharmacy Licensing Exam Questions and Answers- 2nd Revised Edition 3x 10% 9x10" ©, 4. 661. Which of the following class of surface active agents is widely used as a preservativerather than surfactant? Cationic agents Anionic agents Amphoteric agents Tonly Land I only Mand III only all eege 662. The small intestine is the primary site for absorption of an orally administered drugs be- cause Krisman c.Theinhibition of substrate metabolism tends to begin as soon as sufficient concentrations of the inhibitors are reached, Elderly patients are at greater risk from enzyme inhibition reactions. 664. Theelderly are more susceptible to which of the following conditions due to the decline in baroreceptor function ? a Hypertension b Tachycardia c ——_ Bradycardia d-—— Orthostatic hypotension 665. The most widely prescribed ERT for treating menopause symptoms is I Theintestine epithelium is more a. —_Estropipate a. permiable to drugs than the stomach. b. Conjugatedestrogen {°/977" ©. Estradiol I The absorptive area of the intestine is. qd, Medroxyprogesterone greater than that of the stomach 666. As with most menopause symptoms, TIL Thetotal blood flow through the wa ie intestine capillaries is much greater —= ee a. Estrogen deficiency b, _Estrogenexcess a Tonly c. Progesterone deficiency b. Tand only d. Progesterone excess ©, Mand It only < 2 667. ‘The symptoms of BPH are caused by 663. WhichotthefollowingisNOTTRUE pressure exerted by the prostate gland about enzyme inhibition reactions ? eae b. relaxation of bladder muscles a. They occur much more slowly than c. __ shrinkage of the prostate pland induction reactions. relaxation of the prostate gland muscles ‘They are the mostclinically significant typeof interactions. www.pharmacyexam.com ~ 668. The primary cell type responsible for the Jong-term effects of asthma is 82 Reference Gt ‘Questions and Answers- 2nd Revised Edition a b. —_ Basophil c. Tlymphocyte 4. Blymphocyte 669. The major immunoglobulin associated with asthma is a IgA b IgM 1 c eB u a IgG 670. Which of the following factors triggers an asthma attack ? I Tobacco smoke TI Exercise MM — Coldair only Tand II only Hand Ill only All peop 671. Which of the following is the clinical hallmark of asthma ? Sneezing Coughing Wheezing Dozing eee 672. Wheezing is defined a. _apersistent cough or tightness in the chest. b. _ahigh-pitched sound caused by turbulent airflow passing through an obstructed airway. ¢. _adegranuation of eosinophils and release of chemical mediators. 4. abronchoconstriction, swelling, and an excessive mucous secretion in the airway. www.pharmacyexam.com je for Foreign Pharmacy Licensing Exam Krisman 673. Blood dyscrasias that affect all three blood cell lines are defined as: a rombocytopenia b. —Agranulocytosis ©, Leukemia d.—Pancytopenia 674. Which of the following drug-induced hematological disorders are associated with the highest morbidity and mortality rates ? a. Hemolytic anemia b. Leukemia ¢. Aplastic anemia d. granulocytosis 675. Which of the following drugs causes hemolytic anemia in patients with G6PD de- ficiency? 1 Cortrimoxazole 1 Dapsone TM Primaquine a Tonly b — TandIlonly ¢ Hand Ilfonly ds All 676. Which of the following is a sign and symptom associated with drug-induced aplastic anemia? 1 Fatigue Il Frank bleeding a Tonly b. Land Ilonly ¢. Hand Illonly 4 All 83 Reference Guide for Foreign Pharmacy Licensing Exam ‘Questions and Answers- 2nd Revised Edi 677. Apatient’s blood report indicates granu- locytes count of 1000 cells/mm, This condition isdefined as: 1 Granulocytopenia Tl Leukopenia TI Agranulocytosis a. Tonly b. Land I only c. Hand Il only a All 678. The term that normally describes for a // rug that when combined with a protein ren- ders it antigenic is a, Immunophilic b. — Heptane cc. Antiglobin d.— Thromboxatic 679. Which of the following is the most com- mon causative agent for Community Acquired Pneumonia? a. S.pneumonia b.- Mpneumonia c— Saureus 4. Cpneumonia 680. Which of the following is NOT TRUE about Xerostomia ? a It causes dental caries. b, —_Itcauses difficulty in swallowing medications. c. It causes significant distress to patients 4. Tteontributes excessive salivation, 681. Theelderly are more susceptible to which of the following conditions due to a decline in baroreceptor function ? www.pharmacyexam.com a Hypertension b Tachycardia c — Bradycardia 4 —_ Orthostatic hypotension 682. Temazepam and Lorazepam are pre- ferred benzodiazepines for use in the elderly because a They donot undergo phase I reactions. b They do not undergo phase II reactions. c They extensively excrete by first pass ‘metabolisms, d— Theyhave limited first pass metabolism. 683. Amlodipine is classified as: Beta blocker ACE inhibitor Ca-channel blocker Vasodilator aege 684. An amine base that is not metabolized and has a PKa of 8 will be reabsorbed from the renal tubules most quickly if the pH of the urine is adjusted to: 5 6 3 1 aege 0. 685, Ten hours afier 750 mg of adrug is ad- ministered by IV injection, a patient's plasma con- centration is 20 meg/ml. If the half-life of this drug is Shoursand the minimal effective concentration (MEC) is 3 meg/ml, how many hours after the first dose should a second dose be administered? a 10hours b. 22hours ¢. 37.Shours 4. 1Shours Reference Guide for Foreign Pharmacy Licensing Exam ‘Questions and Answers- 2nd Revised Edition 686. Thefollowingtwosystems were prepared without surfactant. System A System B Volume of oil 10mi 100m Droplet of radius 1 mem mem Interfacial tension __80dyne/em $Odyne/em Volume of water 250 ml 250 ml Which of the following statements bout the thermodynamic stability of SystemA is true? Itis as stable as system B b. tis 10 tintes more stable than system, B, cc. It is twice as stable than system B. d. Iti 50 times more stable than system B. 687. Theradioactive decay of Mobidium has arate constant of 5x 10° year. How many years will it take for 90% of the Mobidium initially presentto degrade ?(The amount initially present canbe specified as 100%) a 25 b. 2345 c 3243 a 4606 2 688. A solution contains methyl acetate (0.1M) ~ and sodium hydroxide (0.1M), If the rate con- stant for this reaction at 25°C is 1.082 liters (mole.min), how many minutes will it take for the concentration of methyl acetate to fall 0.09 M? 1.026 23.45 132.15 5.43 ees 689. Thehalf-life of a drug is 12days.A single 7 $00 mg dose of the drug yields an [AUC] value ‘www.pharmacyexam.com Krisman .0f 360 meg/h/m. In micrograms per milliliter, what plasma level will result ata steady state if this product is given twice a day and is 80% bioavailable? a 10 b 45 ec 30 de 690. A 24-hour urine sample is collected from \ apatent whohas ningun diabetes mellitus and stable creatinine level of 2 mg/dL. The sample shows atotal volume of 1400 ml and creatinine concentration of 100mg/dL.. In milliliters per ‘minute, what is the approximate glomerular filtra- tion rate for this patient ? z i ) crpR = Use BO eee 4d 120 if Pepary dco 691. Which ofthe following NSAIDS is indi- cated for the treatment of ductus arteric a. buprofen b.— Indomethacine cc. Rofecoxib d. — Nabumetone 692. Which of the following enzymesisa rate limiting step in glycogenolysis? a, Glucose-6-phosphate b. _ CAMP-dependent proteinkinase c. Glycogen synthase d. Glycogen phosphorylase 693. Which of the following is the most potent ‘estrogen in the human body ? Reference Guide for Foreign Pharmacy Licensing Exam (Questions and Answers- 2nd Revised Edition Estradiol Estrone Estriol Ba Tonly Land Il only Mand III only Al aece |. Which of the following is NOT TRUE ‘about Neonatal tetany? 1 Ttisagenetically fatal disorder. I Ttoccurs in newborns of mothers with hyperparathyroidism. II It disappears as soon as the infant’s parathyroid gland responds normally. a Tonly b — Tand Ionly © Hand {llonly dal 695. Which of the following is useful in treatment of inappropriate secretion of ADH? a Doxycycline b. Demeclocycline e Chlorpromazine 4. Rivastigmine 696. Which of the following has the longest half- life? Ciprofloxacin b.— Clofazimine cc. Sulfadiazine d.—Dicloxacilin 697. The partial supply of Controlled I drugs should be filled within a. 12hours b. 72 hours www.pharmacyexam.com Krisman c. — 48hours d.— 4hours 698. Which of the following drugs may be found under the Schedule II classification? T Fentanyl TT Amphetamine TI — Methylphenidate a Tonly b Land only ¢ Mand Ill only 4 al 699. The DEA requires that the inventory of Schedule II should be done: every week every six months every two years every five years aege 700. The partial dispensing of Schedule III controlled drugs should be done: a. everyyear b. 6 months cc. — everyweek 4. 3months 701. Which ofthe following auxiliary labelsis/ are required before dispensing Biaxin (Clarithromycin) suspension ? 1 Take with food, Donotdiscontinue. TM Store in refrigerator. a Tonly b ——_ TandIonly c Wand II only dll 86 Reference Guide for Foreign Pharmacy Licensing Exam Questions and Answers- 2nd Revised Edition 702. A patients taking Precose (Acarbose), 100 mg three times a day. The pharmacist has to advise the patient to regularly check his a AST b scr c INR der 703. An example of HMG-COA reductase enzyme inhibitoris/are Simvastatin TL. Pravastatin T. — Lovastatin a Tonly b.— Tand Ifonly ¢— TMand If only a all 704, Which of the following drug(s) is/are used for obsessive compulsive disorder? 1. Clomipramine |. Imipramine TM Amoxapine only Land I only Mand lLonly All pore 705. A patient is using an inhaler for the first time. As a pharmacist, you would advise himto 1. Hold the Aerochamber and shake vigorously 304 times. I __Breathein slowly and deeply through the mouth until you have taken a full breath, I. Hold breath for Sto 10 seconds and repeat steps. www.pharmacyexam.com Krisman a. Tonly b. — TandIonly c. MandIIfonly a all 706. KetorolacTromethamine can be. given ont L Orally em i IM. mh LV. a Tonly b. Land Ionly c. Wand If only a all 707. Which of the following is commonly used as ahousehold oxidizing agent? a. Formaldehyde b. _Benzalkonium chloride c. Hydrogen peroxide 4. Ethylene oxide 708. ‘The primary mechanism of action of Esomeprazole is H, receptor antagonism 5-HT, receptor antagonism D, receptor antagonism Gl. proton pump inhibition aoge 709. Erythromycin is a choice of drug for treatment of a. Onychomycosis b. Legionnaires’ Disease c. Herpes simplex a HIV 710. Which ofthe following statement(s) about ‘Talwin NX is/are true? 1 ‘The active ingredients of Talwin NX are Pentazocine and Naloxone HCI. 87 Reference Guide for Foreign Pharmacy Lice1 Questions and Answers- 2nd Revised Edition Tl. Itisintended for LV. and Muse. Ml, Naloxone HCI (0.5 mg) has profound antagonist activity when given orally. a Tonly b. — Tand Monly Hand Hf only da Al TIL. All of the following drugs may raise the blood serum concentration of Theophyl- line EXCEPT a. Allopurinol b. Primidone c. ‘Cimetidine d. Ciprofloxacin 712. Which of the following drug(s) is/are indicated for treatment of chemotherapy in- duced nausea and vomiting? Ondansetron Granisetron Dolasetron RRO a Tonly b. Land Monly c. — Tland [only a All 713. Mr. Keit is taking Diphenhydramine elixir to treat his allergy problem. Suddenly one night he suffers from flushing, chrobbing headaches, breathing difficuity, nausea, vomiting, weakness, and blurred vision, These symptomsare because of a. Lorazepam b.—Protriptyline ¢. Chlorpropamide d.— Diphenhydramine www.pharmacyexam.com ¥x ye ing Exam Krisman 714. Microorganisms that establish permanent residence without producing disease are known as: a. Normal flora b. —Transientflora c. Opportunistic pathogen 4. Microbial antagonism 715. Allof he following drugs are serotonin reuptake inhibitors used in the treatment of de- pression EXCEPT a. Venlafaxine b. Paroxetine cc. Fluoxetine da. ‘Tranylcypromine 716. Otitis mediais generally caused by 1H. Influenza HS. Pneumonia I. — M.Pheumonia a Tonly b. Land Ionly ce. Wand Ill only a All 717. Otitis externais generally caused by: a, S.aureus b. —_ Sepidermis c. aeruginosa 4. Spyrogen 718. Mrs. Rey wants touse an Estraderm sys- tem for the first time. She comes to the pharma- cist and asks how to use the transdermal system. ‘The pharmacist may counsel her ofall of the fol- lowing EXCEPT a, Thepatch should be placed on aclean, nonhairy and dry area of skin. 88 Reference Guide for Foreign Pharmacy Licensing Exam Questions and Answers- 2nd Revised Edition b. Thesite of the application must be rotated at least every week The breast is the best site for the application of the patch. @ ——_Thesystem should be applied immediately after opening the patch. 719. Which of following about Didanosine isi are true? 1. Tthas a more prolonged duration of action than Retrovir, with least bone marrow suppression property. Il _ The percentages of patient survival with Didanosine is higher than with Retrovir, I. Antacids may reduce or protong the absorption of Didanosine, so an interval cof atleast 2 hours berween administration of Didanosine and antacids istequired. a Tony b. — TandIonly c. IandIIltonly a Al 720. Cold sores or fever blisters are normally caused by: 2 Bs b. -HSV2 c. Varicella zoster d.—Variola minor 721, Which of the following antibiotics should the avoided by patients who are suffering from GOPD deficiency” a Tetracycline b.— Penicilin www.pharmacyexam.com oe ee Krisman. c.— Solfonamide d.—— Cephalosporin 722. ‘Trachomaisnormally caused by: Tinca capitis Sporotrichosis, Corynebacterium diptheria ‘Chlamydia trachomatis apge 723. Which of the following drugs should be used for vitamin K deficiency? a. Exgocalcifero} b. —Phytonadione cc. Ascorbic acid Pyridoxine 724. Mr.Xissuffering from hypersensitivity reactions with Pentazocine. Which of the follow- ing drug(s) should be avoided by Mr. X because ofhisallergy? ‘Talacen 1 TalwinCompound Mm. TalwinNX Tonly Land I only Mand Ilonly All eese ‘725, Mr Keit stats to takea Pancrelipase cap- sule for treatment of nutritional disorder. After ‘one week, he lost control of his blood glucose level. When he went to the pharmacy and asked the pharmacist about his problem, the pharmacist ‘counseled him that 1. Panerelipase enzyme increases the conversion of the complex carbohydrate wwaglucose. 89 Reference Guide for Foreign Pharmacy Licensing Exam ‘Questions and Answers- 2nd Revised Edition Il. Panerelipase enzyme generally triggers the blood glucose level by inhibiting insulin secretion. TI. Pancrelipase enzyme increases insulin secretion. a. Tonly b. Land IT only c. Hand Il only a All 726. Which of the following medication(s) should be avoided with Ketorolac? L _Plicamyein I. Valproic acid TL Moxalacta a Tonly b. Land only ¢. Hand Il only a All 727. Which of the following drug(s) is/are An- giotensin II receptor Antagonist(s) ? 1 Valsartan Losartan I Irbesartan a Tonly b Land only c TMandilfonly dll 728. Which ofthe following informations sare true about Infectious Mononucleosis? 1. Ttiscaused by Epstein Barr virus. 1. tis characterized by enlarged and tender lymph nodes, an enlarged spleen, fever, sore throat and headache. www.pharmacyexam.com Krisman IIL _Itiscommonly spread by sneezing. a Tonly b. Land only c. MandilTonly a All 729. Which of the following TCA is used in the treatment of nocturnal enuresis? Doxepin Imipramine c.—Protriptyline d. Amoxapine 730. Corlopam (Feoldopam) is indicated for short-term (up to 48 hours) management of se- vere hypertension, Its classified a(n) 2 Allpha-1 receptors agonist b —_Beta-2 receptor agonist © Dopamine-DI receptor agonist d— Histamine-H1 receptor agonist 731. Deferoxamine mesylate is administered by 1 LM.route T. — LV.route MI. S.C. route a Tonly b. Land ITonly c. Tad Ilonly a4 All 732. Metoclopramide produces its antiemetic effect by 1. Inhibiting stimulation of the chemoreceptor trigger zone. IL Increasing GI. motility. TIL Increasing the rate of GI. emptying, 90 Reference Guide for Foreign Pharmacy Licensing Exam Questions and Answers- 2nd Revised Edition a Tonly b — TandIfonly c. — Tand JIT only da All 733. Burkit’s lymphoma is caused by: a S.aureus b. — Tppallidum c. Epstein Barr virus 4. Toxoplasma gondii 734, Allofthe following drugsare useful inthe treatment of status epileptics EXCEPT a 50% dextrose solution b. Diazepam cc. Phenytoin 4. Tiiazolam 735. Which of the following about Trimethobenzamide is tue? L.__Itisindicated for control of nausea and vomiting. I. Itisavailable in caplet, injection and suppository form. TI. NoAB or ATrating is listed for ‘Trimethobenzamide. a Foaly b. and Ilonly ce. fland Monly da All 736, Which of the following is NOT TRUE about Vibramycin? I. _Itisindicated for the treatment of SIADH. IL Tecan be safely used in renally impaired patiens. www.pharmacyexam.com, Krisman II. Itmay be taken with food or milk to reduce GL. irritation. Tonly Tand Il only Mand Ionly All pegs 737. Sunscreen products should cover a 500nm UV light b. 400. nm UV light cc. 320nm UV light 7 100 nm UV light 738. Which of the following is known as the pacemaker of the heart? a SAnode b. AV node c. Purkinje fibers 4. Tricupsid valve 739, Which of the following is/are true about ‘Zofran? I. _Itisindicated for the prevention of cancer chemotherapy induced nausea and vomiting Il {tis available in tablet, injection and oral solution forms. IIL, Theactive ingredient of Zofran is Ondansetron HCI. only Tand Ionly Mand IM only All eege 740. Which ofthe following hormones is se- creted by the posterior pituitary gland? o1 Reference Guide for Foreign Pharmacy Licensing Exam ‘Questions and Answers- 2nd Revised Edi a. Prolactin b. Follicle stimulating hormone cc. Vasopressin 4. Lateinizinghormone 741. Which of the major serum proteins is in- volved in the protein binding of drugs ? a. Allpha-1 acid glycoprotein b. Lipoprotein Albumin a Beta-1 acid glycoprotein 742. Which of the following is/are TRUE, about Advair Discus ? 1 Itisacombination product of Fluticasone and Salmeterol. a ‘The normal therapeutic recommended dose of Advair is twice a day. I Itshould be immediately used for prevention of acute attacks of asthma. a. Tonly b. — TandIfonly c. Mand lfonly a All 743. Allofthe following drugs are useful dietary supplement products of pancrelipase enzymes EXCEPT a Cotazym, b. — Pancrease. © Zymase. d.— Pulmozyme. 744, Fameyclovir is generally indicated forthe treatment of aS. Aureus b.— C.Albican www.pharmacyexam.com Krisman c. M.Leprae & —-H.Zoster 745. Whichof the following isfase true about Pentam-300? L Itis mainly indicated for treatment of P. Carini Pneumonia. TI. Severe hypotension will be observed when given by LM.or LV. TIL Prescriptions for Pentam-300 can be filled by NebuPent. a Tonly b. Land Ionly e. THand IMlonly a All 746. Mesalamine generally acts by a. Binding to 30, ribosomes. b. _ Binding to 508 ribosomes. ¢. Local bactericidal effects oncolon bacteria, 4d. Inhibiting prostaglandin synthesis, 747. Thenormal range of prothrombin time is a 1 to 5 seconds b, 12 to 15 seconds ¢. 2010 30 seconds d. 30 to 60 seconds 748. The body mass index is calculated by: a. Weight (kg) /body surface area b. Weight (Ib) /height (inches? c. Weight (kg) / height (meter)? 4 Weight Ib) /height (meter) 749. Renal clearance of phenobarbital can be increased by administering 92 Reference Guide for Foreign Pharmacy Licensing Exam Questions and Answers- 2nd Revised Edition a Sodium bicarbonate b. — Sodiumcitrate Magnesium citrate 4. Potassium chioride 750. Rifabutin doses should be reduced by ‘75% from the recommended dose when coadministered with a Lopinavir b. — Gemfibrozil < — Glipizide d— Olanazapine ‘TSA. Severe headaches are commonly asso- ciated with a Isosorbide dinitrate b. — Furasemide Bumetanide d. Propranolol 752. Posicor (Mibefradil) i ‘treatment of indicated for the CHF Hypertension Arthritis BPH epee 753. Which of the following is/are true about Accolate? 1 __Theactive ingredient of Accolateis ‘Zafitlukast. Itacts by antagonizing LTRA- receptors which are responsible for the telease of SRSA. II. _Itisnota bronchodilator and should not be used for treatment of acute attacks of asthma. www.pharmacyexam.com, Krisman a only b.— TandIonly Mand If only a all 754, Allofthe following drugs areinsdicated in the treatment of organ transplantation as immu- nosuppressive agentsEXCEPT a. Azathioprine b. Cyclosporine ©. ‘Tacrolimus 4. Rocuronium 755, Which of the following is/are first-line antihypertensive agent(s)? 1 HCTZ I. Atenolol IL. Captopril a Tonly b. and Itonly ©. Hand Ilfonly a. All 756. ‘The probable mechanism of action of Losartan is a. By inhibiting the enzyme whichis responsible for conversion of Ace Tin Ace Il. b. By vasodilation of peripheral arteries. By blockingAlpha-I receptors. By blocking AT type II receptors. 757. Skin protection factor (SPF) must be 1. Sormore, but less than 14 I 10ormore, but not less than 14 I, 1S ormore a Tonly b. Land I only Wand Wlonly a Al 93 Reference Guide for Foreign Pharmacy Licensing Exam Questions and Answers- 2nd Revised Edition 758. Capsofungin is indicated to treat infec tions caused by a. Herpes virus b. — S.pneumonia ce. Afungi d—V.cholera 759. The chance of hypoglycemia is the least when therapy is maintained with Acarbose ‘Metformin ‘Troglitazone only Land I only Mand I only All a b. c 4. 760. Which of the following is/are bulk laxatives ? 1. Metamucil I. — Citrucel TL — Miolan a Tonly b. and ITonly c. Tand Ilfonly a aD 761. The primary mechanismof action of Pan- toprazole is a a. H,receptorantagonism b, S-HT, receptor antagonism ©. D, receptor antagonism 4. Gl. proton pump inhibition 762. Which of the following drug(s) should be avoided during Rezulin therapy? www.pharmacyexam.com Krisman 1. Cholestyramine I. Colestipol ML. —Terfenadine a. Tonly b. — TandIonly c. Mand {ifonly da All 763. All of the following mentioned drugs produce cardiac toxicity (like cardiac arrhyth- ‘mi) when given in combination with Ketoconazole EXCEPT a. Astemizole b. —Terfenadine cc. — Loratadine 4. Cyclobenzaprine 764. Which of the following is/are true about NSAIDs? : NSAIDs are indicated for treatment of analgesia, dysmenorrhea and bursitis. I. GL.ulceration and bleeding have been seported with the use of NSAIDs. III. Any OTC NSAID should not be used more than 15 days for pain or 10 days forfever. a Tonly b. Land Ilonly Mand IIfonly do All 765. Which of the following about Lovenox is/are true? 1 Theactive ingredient is Enoxaprine Na. 1. Itisalow molecular weight heparin. 94 Tris available ina 30 mg/0.3 ml injection, Tonly Tand Tl only Mand lonly All 166. The major side effect of Cylert (Pemo- line) that restricted its use in individuals is Renal failure Heart failure Liver failure Brain failure poge ‘167. The probable mechanism of action of Ga- bitril (Tiagabine) is Alpha-1 receptors blockage Enhanced activity of GABA. Beta-I receptor agonists Serotonin SHT, receptor blockage aoge 768. Theactive ingredient of Combiventis/are 1 Ipratropium Br IL Albuterol S04 TI. — Metaproterenol a Ionly b. Land Ionly cand lLonly d. All 169. Which of the following drug(s) is/areAn- giotensin type I receptor antagonist(s)? 7 Losartan Tr Irbesartan + HCTZ TM Zileuton a. Tonly b. Land I only www.pharmacyexam.com ence Guide for Foreign Pharmacy Licensing Exam ions and Answers- 2nd Revised Edition Krisman ¢. MandTlonly a All 770. Which of the Insulins has the shortest du- ration of action and the fastest onset of action ? a. Humulin-R b. Lantus <. Humalog 4. Novolog 771. The strength of folic acid in OTC vita- ins should not exceed ao 1somg b 10 mg c 04mg d 10 meg 772. Before initiating therapy with Paclitaxel, itis recommended that the patient be pretreated with 1 Corticosteroid I Diphenbydramine HI, H, Antagonist a Tonly b. — TandIfonly c. Mand Ml only a All 773. Which of the following drugs is contrain- dicated in the treatment of B.PH.? 1 Diphenhydramine Tl __ Phenylephrine/PPA/Gauifenesin M —Atropin Tonly Vand Honly Mand Ionly All eese 95 Reference Guide for Foreign Pharmacy Licensing Exam ed Edition Questions and Answers- 2nd Re 774, Sumatriptan is mainly indicated for the treatment of a. Cancer therapy induced nausea and vomiting b. Migraine c. Depression d. Seizure 775. Methicillin resistant infection would be treated by Penicillin G Cephalexin ‘Vancomycin Ceftriaxone pore 716. Which of the following antihypertensive agents helps patients on diabetic treatments? a Verapamil b. Enalapril c. _ Hydrochlorothiazide d.—Amiloride 777. The active ingredient of Compound W is a Sulfur b. Aspirin ©. Salieylicacid d. Coal tar 778. A patient is on Antabuse for his treat- ment of alcohol withdrawal. Which of the following product(s) should not be taken with Antabuse ? L ‘Theophyllin Na glycinate elixir I Theophyliinelixir Il. Theophyllin tablet a Tonly b. Land only Mand Ill only a All ‘www.pharmacyexam.com 779. A depressed person with anxiety disor- der should be treated with a. Doxepin b. Nortriptyline c. Fluoxetine d. Amitriptyline 780. ‘The mechanism action of Fluoxetine HCL isthe reuptake inhibitor of norepinephrine. reuptake inhibitor of epinephrine reuptake inhibitor of serotonin. reuptake inhibitor of dopamine. eege 781. Ondansetron is mainly indicated for the: a. Prevention of hypertension. b. Prevention of nausea and vomiting induced by cancer chemotherapy. ¢. Prevention of migraine attack. 4. Preventionof angina, 782. All of the following drugs should be avoided with KetorolacEXCEPT a. Meclizine b.— Cefotetan ©. Goldcompound 4. Methotrexate 783. Which of the following hormone is se- creted by the kidney in response toa reduction in the amount of oxygen that reaches tissues? a. Calcitonin b. Erythropoietin ce. Insulin d. Melatonin 784. Carbamazepine is mainly indicated for 96 Reference Guide for Foreign Pharmacy Licensing Exam Questions and Answers- 2nd Revised Edition a Trigeminal neuralgia & Arthythmia © Hypertension Depression 78S. Erythromycin is the preferred agent for treatment of L— Legionnaire’s disease I. Mycoplasma Pneumonia Il. Campylobacterial infection a Tonly b. Land Tonly ec. Wand Ill only a Al 786. Alof the following drugs act as anti- hyperlipidemic agents by inhibiting HMG- COA reductase EXCEPT a. Lovastatin b. Simvastatin c Fluvastatin, d.— Clofibrate 787. Afierinitiationof therapy, Mr. Teddy sud- denly suffers from spasms of the face and neck. ‘Which of the following drug(s) should be given to alleviate the above symptoms? I. Diazepaminjection TW. —Diphenhydramine 1. —_Benztropine a. Tonly b. Land Ionly © and 1lfonly a AL 788. Cholestatic hepatitis is commonly asso- ciated with a Erythromyein www.pharmacyexam.com Krisman b. __Exythromycinethylsuccinate c. Erythromycin stearate 4, Erythromycin estolate 789. Which of the following prostagiandinsis ‘a potent mediator of asthma attacks? a. PGD2 b. PGE2 ce. PGF2 ad PGIZ 7190. The purity of Insulin can be measured ‘Content of Proinsulin Content of Acid buffer Separation of Dextroinsulin Degradation of Insulin at elevated temperatures 791. Nirs, Xisa pregnant woman. Which of the following antithyroid agents should be sug- «gested by her pharmacist? L PTU 1. PIMASyrup TH, Methimazole a Tonly b. Land itonly ¢. Hand {tfonly d. All 792. Which of the following drugs is/are dopamine receptor agonists ? 1 Bromocriptine . — Pergolide UL. —Tranyleypromine a. Tonly b. Land only c. HandIMonly a All 97 Reference Guide for Foreign Pharmacy Li Questions and Answers- 2nd Revised Ed 793. The patients should avoid sunlight when he or she ison Hydrochlorothiazide Thioridazine Tetracyclin HAT a Tonly b. Land Ionly cc. Hand Ilfonly a all 794, Which of the following is NOT TRUE, abouthistamine? a. Itinereases the contraction of smooth muscles, b. _Itdecreases force of contraction and rate of heart. ¢. __Itstimulates gastric acid secretion, 4. Itisreleased in large amountsafier skin damage. 795. ‘The normal therapeutic blood serumcon- centration of Phenytoin is 101020 mg\ml 101020 meg\ml 301050 meg\ml 101020 meg\ml esse 796. Allofthe following are adverse effects of goldcompoundsEXCEPT a Diarrhea b. Abdominal pain c. Stomatitis 4. Lipoatrophy 797. Which ofthe following antihistamine(s)is! are nonsedative antihistamines ? L— Cefirizine UH Terfenadine Ti. Astemizole www:pharmacyexam.com Krisman Tonly Tand only Mand Ionly all aege 798. The minimum weighable quantity for class-A prescription balance is, a 100mg b. 80mg ce. 60mg ad 120mg 799. Which of the following is/are true about Bupropion? I. _Itis presumed to act on dopaminergic and noradrenergic pathways involved in nicotine addiction and withdrawal. I __Ithelps reduce the urge to smoke. IIL. _Ithelps reduce nicotine withdrawal symptoms a Tonly b. — TandIonly c. Hand MTonly da All 800. Troglitazone is mainly indicated for treat- mentof a. Hypertension b. Diabetes c. Hyperthyroidism, @ Car 801. Mr.Zecis taking Verapamil SR 240mg for treatment of his blood pressure. The pharma- cist may tell him that he must be careful using the following drugsEXCEPT a. Atenolol yr Reference Guide for Foreign Pharmacy Licensing Exam Questions and Answers- 2nd Revised Edition Digoxin c. _ Disopyramide d.——tndomethacin 802. Which of the following drugs should not be used for more than 10 days for allergic con- Jjunctivitis? Ketorolac Loteprednol Azelastine Tetrahydrazoline aoge 803. Thedeficiency af which of the following may increase the chances of bleeding? a. Red blood cells b. White blood cells c.Flymphocytes d. —_Thrombocytes 804. Which of the following drug(s) is/are re- sponsible for pulmonary dysfunction? 1 Bleomycin I Bromocriptine TL. Pentamidine a Tonly b. —_TandIonly ¢. Tan only d. Al 805. Which ofthe following Ca-channel block- ersis useful in the treatment of cerebral spasm? Amlodipine Nifedipine Nimodipine Isradipine apge 806. ‘Trimethobenzamide should be avoided inpatients who are suffering from www.pharmacyexam.com Krisman Ulcers Reye’s syndrome Hypertension Depression 807. Which of the following is/are Bile-acid binding resi Bege I Cholestyramine MH —— Colestipol MM Colesevelam a Tonly b. Land Honly c. Mand If only a Al 808. Which of the following may be damaged in Multiple sclerosis? a, Nephron b. Myelin sheath c Parietal cells 4. Bowmancapsule 809. All of the following is/are true about Fentanyl TOSEXCEPT a, Fentanyl TDS system is available in 50,75 and 100 micrograms of strength, b. __Itiscontraindicated to use for ‘management of mild to intermittent pain, ¢. AS micrograms per hour dose should be considered as a safe dose for initiation of therapy. 4. Fentanyl patches should notbe administered to children under the age of 12 or adults under the age of 18 ‘whose weight is less than 110 Ibs. 99 Reference Guide for Foreign Pharmacy Licensing Exam Questions and Answers- 2nd Revised Edition 810. Furosemide isavailablein L Tablet I. Oralsolution I. Injection a Tonly b Land I only © Hand If only a All 811. Which of the following NSAIDsis a pro- dmg? Diclofenac Ibuprofen Diflunisal Indomethacin aege 812. Valproic acid mainly acts a. byincreasing the concentration of GABA. b. by decreasing the concentration of GABA. c. by increasing sodium influx in the brain. d. __bydecreasing firing of chloride ions in the brain. 813. The Food and Drug Administration di- vided drugs according to their potential to cause birth defects in 5 different categories. Which. of the following categories indicates the highest. risk to the developing fetus? pose xawD 814. Which of the following sedative hypnotic agents is useful as an antiemetic agent for treat- ‘ment of cancer chemotherapy induced nausea and vomiting? www.pharmacyexam,com Krisman a. Oxazepam b. Clonazepam c.— Prazepam 4. Lorazepam 815. Which of the following glands secretes Melatonin? a. Pituitary b. Thyroid cc. Pineal d. Pancreas 816. A patients has “High Cholesterol” if his total serum cholesterol value is greater than a 120 mg/dl b. 360. mg/dl c. 180mg/dt a 240 mg/d 817. The primary mechanism of action of ‘Terazosin is a(n) beta-1 blocker alpha-1 blocker beta-2 blocker ACE inhibitor apse 818. A patientis allergic to Amitriptyline. Which of the following seizure drug(s) should he avoid because of his allergy problem? 1 Carbamazepine I. Cyclobenzaprine MI —Valproic acid Na a Tonly b Land only © Mand I only ds All 819. Ketonuriais normally found in patients with/afier: 100 Reference Guide for Foreign Pharmacy Licensing Exam Questions and Answers- 2nd Revised Edition L Diabetes mellitus I, Severevomiting I. Starvation a Tonly b. — LandITonly Hand Ifonly a All 820. Which of the following is/are adverse effect(s) of Phenytoin ? 1. Gingival hyperplasia UL Lupus erythematosus tL. Ataxia Tonly Land only Mand Iitonly All aege 821. Hydrochlorothiazide causes all of the following EXCEPT Hypokalemia ‘Hypercalcemia Hypouricemia Hyperglycemia Bose 822. Meniere’s disease is characterized by: Episodes of deafness Vertigo Buzzingiinthe ears AEE Tonly Land Honly Mand If only All aoge 823. A deficiency of which of the following may lead to goiter? a. Magnesium wwi.pharmacyexam.com Krisman >. Aburriram c. Iodine d. Fluorine 824. Which of the following antifungal agent(s) {fare useful in the treatment of oral candidiasis? L Itraconazole TL Clotrimazole JIL Nystatin Sonly Land Il only Hand Hf only All eoce 825. Patients allergic to Cyclosporine need toavoid 1 Sandimmune MT — Gengrat i Remeron a. Tonly b. Land only ©. Mand Iif only a all 826. Which of the following is NOT TRUE, about anorexia nervosa? a. Itisagenetic disease characterized by severe weight loss. b. _Itiscommonly reported in adult females. c. — Thereisanexcessive use of laxative or emetic agents, d. Fluoxetine is indicated for the treatment, 827. Zidovudine-induced anemiais best treated 101 Reference Guide for Foreign Pharmacy Licensing Exam Questions and Answers- 2nd Revised Edition 1 EpoteinAlfa T. Filgrastim TI Sargamostim a Tonly b. Land Tlonly cand If only a All 828. The overdose symptoms of Propoxyphene is/are I. _ Respiratory depression with cheyne stokes I, Hypoxia, pinpoint pupil construction and pulmonary edema, TL Circulatory collapse. a Tonly b Land I only c — HandIllonly dal 829. Mrs. X comes into the pharmacy to find the drug she used in Europe. Pharmacists may suggest al ofthe following reference materials EX- CEPT a. Matriandale Extra Pharmacopoeia b. Index Nominum ¢. United State Adopted Name 4. Dmug’s Fact and Comparison 830. Whichof the following about Methyldopa is/are true ? I Before the treatmentis initiated, itis advisable to do a blood count and peri- odic blood counts should be done thereafter. Tl — Hemolytic anemia occurs during therapy; remit promptly upon discontinuation of the drug, www.pharmacyexam.com Krisman TI Prolonged:herapy with Methyldopa ‘may result in a positive Comb’s test. a. Tonly b. Tand only c. Wand IIT only a At 831. Hemoglobin tests measure | gm of He- moglobin per a. 1 ml volume of whole blood b. 1 dL volume of whole blood c. I gmvolume of whole blood d, liter volume of whole blood 832. All of the following statements about Skelid (Tiludronate Na) are true EXCEPT a, Itsactiveingredientis Tiludronate sodium, useful in the treatment of Paget’s disease of bone. b. __Itisavailable in once a day dosing ¢. The serum alkaline phosphate level greatly reduces afterinitiation of therapy. 4. itcanbe safely administered in renally impaired patients. 833. Allofthe following terms areelevated dur- ing acute attacks of asthma EXCEPT a TLC b. FRV c RV ad FEV-1 834. A female with her three-year old child ‘comes to the front counter of the pharmacy and asks for any nausea preventing medicine for her son. She says that her son has suffered from nausea and vomiting for the past three days. The phar- ‘macist may recommend: 102 Reference Guide for Foreign Pharmacy Licensing Exam Questions and Answers- 2nd Revised Edition a Emetrol for controlling nausea and vomiting for three to four times day. b. —Toadmitherson immediately tothe hospital. Plenty of fluid. 4d. Overthe counter Meclizine. 835. A malignantdisease in which the bone ‘marrow may produce excessive amounts of leu- kocytes is known as: a Polycythemia vera b. Thrombocytopenia e. Anemia @ Leukemia 836. Whichof the following is/are true about PEFR measurement? 1. The patients should measure and record PEFR twice daily for 2 weeks. I. ‘The PEFR should be measured in the morning immediately upon rising and 10to 12 hours after the first PEF ‘measurement. II. The“personal best” PEFR is the highest PEFR found in a 2 week period. a Tonly b Land IT only © Tand IIT only dal 837. The onset of action of Insulin Lisprois a Sto 15min, b. 301060 min, ce. 810 1Ghours. dt 2 hours. www:pharmacyexam.com Krisman 838. Which of the following antihistamines is specifically used in the treatmentof vertigo? a. Loratadine b.— Clemastine ce. — Cetirizine d.— Meclizine 839. The probable mechanism of action of Methotrexate is a(n) a. Folicacid antagonist b. Prostaglandin Antagonist cc. Alpha-1 blocker d.—Cyclooxygenase enzyme inhibitor 840. Night blindness is reported due to dietary deficiency of: a. VitaminC b.— VitaminA c. VitaminD a VitaminE, 841. The process by which the hypothalamus stimulates the pituitary to release particular hormones in response to a decrease of the hormone in blood is defined as: Positive Feed-Back Mechanism Negative Feed-Back Mechanism Releasing Factor Stimulation Factor Specific Hormone Stimulation aoge 842. Which ofthe following drugs is useful in the treatment of Cystic Fibrosis? a Ceredase b.— Pulmozyme cc. Protropine d—— Nutropine 843. Which of the following is NOT a phar- macological action of Thyroid hormones? 103 Reference Guide for Foreign Pharmacy Licensing Exam Questions and Answers. 2nd Revised Edition a. __Itincreases oxygen uptake, BMR, and calorie production. b. _Itstimulates carbohydrate and protein metabolism. . __ Ttreduces serum concentrations of calcium by diposing blood calcium into bone. dTtincreases heart rate. 844, Which of the following drug(s) inhibit SRSA? 1 Zafirlukast I. Zileuton I. Cromolyn Na a. Tonly b. — TandITonly c. MandTIfonly a All 845. Which of the foliowing benzodiaz- pines has a prolonged duration of action? Alprazolam b.Triazoiam c.— Plarazepam 4 Oxazepam_ 846. Saquinavir should be stored in the re- igerator; however, once brought to room tem- perature itcan be used out within a day b.— Lweek c. 3months a Thour 847. Inmovable joints, which of the following serves.as a lubrication? www.pharmacyexam.com Krisman a Tendons b. Ligaments ¢. Synovial fluid d.— Microfilaments 848. The effectiveness of Warfarin therapy can be monitored by a. Bleedingtime b. Clotting time c. Prothrombin time d. Anticoagulant plasma concentration 849. Which of the following is NOT a phar- macological action of insulin? @. _Ttincreases utilization and oxidation of sugarin the tissues. b. _Itstimulates transports of glucose into cells, €. _Itstimutates breakdown of glycogen in muscles and the liver. 4. Itstimulates protein synthesis and growth. 850. Dryness of mouth, increased thirst, regular heartbeat, muscle cramps or pain, nau sea and weakness are signs of a. Digitalis toxicity b. — Hydrochlorothiazide toxicity ©. Bisacodyltoxicity 4. Diazepamtoxicity 851. Allofthe following factors stimulate i sulin secretion EXCEPT: a. Glucose b. — Growthhormone c. Glucagon 4. Starvation 104 Reference Guide for Foreign Pharmacy Licensing Exam Questions and Answers- 2nd Revised Edition 852. Digitalis glycoside increases: a. Influxofcalcium b. —_Influxof potassium ¢.—_Effluxofsodium 4. Influx of sodium 853. Which of the following drug(s) is/are indicated for treatment of accidental overdose poisoning of drugs? Charcoal plus IL Actidose with sorbitol TM —Actidose Aqua a Tonly b. — TandIfonly «Hand Ifonly 4 AML 854. Calcium channel blockers are indicated for the treatment of angina because: 1 They decrease in myocardial contractility IL They reduce oxygen consumption of the heart. IIL. They reduce coronary blood flow by constricting coronary blood vessels a. Tonly b. Land Ionly ¢. HandIfonly a All 855. Whichof the following is less likely tore- absorb from renal tubules? a. Glucose b. Water ce. VitaminA, a Sodiumions www.pharmacyexam.com Krisman 856. Apatienthas taken DiaBeta, (Glyburide) 2.5 mg for 6 months, He is recently diagnosed with hyperglycemia. Which of the following drugs is responsible for thiselevated level of glucose ? Nifedipine extended release Omeprazole Hydrochlorothiazide Amitiptyline 857. Which of the following is NOT asideef- fect of vasodilators? a. Hypotension b. Tachycandi Edema d. Weight loss 858. Which of the following drug(s) have been reported to interact with Cimetidine? 1 Maalox I. Ketoconazole TI. Theophylline a Tonly b. Land only ©. Mandilonly dal 859. Which ofthe following ca-channel block- ershas the least incidence of reflex stimulation of the sympathetic nervous system? 1 Amlodipine TL Nifedipine MI. Isradipine a Tonly b. Land Ionly . Hand{Ionly d. All 105 Reference Guide for Foreign Pharmacy Licensing Exam Questions and Answers- 2nd Revised Edition 860. Whichofthe following is NOT presentin lymph fluid? a. Lymphocytes b. Urea c. Platelets, 4. Creatine 861. Hepatic toxicity is more common with 1. Iproniazid T. —Phenelzine M1, Tranyleypromine a Tonly b. — TandIlonly cc. and Ill only da All 862. The principal function of the lymph node isto: a, Providemutrientstocells. b. Remove waste material from cells. c. Protect cells against microbes and foreign particles. 4. Synthesize RBC. 863. Methylphenidate SR is generally indicated for treatmentof a. Parkinsons b. — Weightreduction c. Attention deficit syndrome 4. Painreliever 864. Which of the following drug(s) is/are carefully prescribed with Selegiline? 1 Meperidine TL Fluoxetine I. Tyramine www.pharmacyexam.com Krisman a Tonly b.— TandIonly c. Hand Ilfonly a All 865. Which ofthe following drugs was first ap- proved for treatment of HIV? a Saguinavir b. Zalcitabine ©. Didanosine 4. Retrovir 866. Flushing isa major complication of Nia- cin therapy, and can be prevented by 1 Taking Niacin with food. I Initiating therapy with a low dose of the drug. IM Taking aspirin 30 minutes priorto Niacin. a Tonly b —_TandITonly © MandIlfonly a all 867. Theactive ingredient of Humibid LA is a Dextromethorphan b. Codeine cc. Guaifenesin 4. Benzonatate 868. Whenasubject changes the posture from the lying down position to the upright, there isa reflex hypotension called: Systematic hypotension b. _Asymptomatichypotension ¢. Postural hypotension 4 Aortic hypotension 106 Reference Guide for Foreign Pharmacy Licensing Exam Questions and Answers- 2nd Revised Edition 869. Which of the following drugs is NOT an enzyme inhibitor? Cimetidine Fluoxetine Carbamazepine Ciprofloxacin pegs 870. The serum concentration of which of the following neurotransmitter(s) isare increased in pheochromocytoma ? 1 Epinephrine I Norepinephrine Ml, Tyramine a Tonly b. — TandIonly c. HandIlfonly all 871. Which of the following is a reversible, nonselective MAO inhibitor ? a Colesevelam b. Linezolid cc. Fexofenadine 4. Lopinavir/Ritonavir 872. Which ofthe following drug(s) should be avoided with Digoxin? 1 Erythromycin I Quinidine Il. Colestipol a. Tonly b. Land only c. Hand Ill only dll 873. CalciumAcetate is indicated for treat- mentof www.pharmacyexam.com Krisman a. ‘Hypocalcemia b. __ Hypophosphatemia ©. Hypervaloemia d. —_Hyperphosphetamia 874. The principal function of bile salt isto: metabolite carbohydrates emulsify fats inhibit gastric acid secretion and motility provide alkaline pH eese 875. Thenormal therapeutic range of activated partial thromboplastin ime is a. 12 to 14 seconds b. 201030 seconds 351045 seconds d. 1 to5 seconds 876. Vancomycin Enemais indicated for treat- ‘ment of infections caused by: a C.difficile b. C.tetanus c. — Paeruginosa 4. E.Coli 877. Thenormal serum plasma concentration of Potassium is a 1010.20 meq/L. b.-3.5t05meg/L . 4. t0.40 meq/L. > 120 meq/L 878. Which of the following isa short-acting barbiturate? a. Phenobarbital b. Amobarbital cc. Butabarbital 4. Pentobarbital 107 Reference Guide for Foreign Pharmacy Licensing Exam Questions and Answers- 2nd Revised Editi 879. Ritonavir oral solution should be avoided with: a Metronidazole b __Diphenhydramine Amitriptyline Alprazolam 880. Clorazepate SD is indicated for treat- mentof a. Hypertension b. Anxiety c. Insomnia da BPH 881. Administration of which of the following drugs requires caution when using with Fenofi- brate? Erythromycin aoge 882. Which of the following drugs should be carefully prescribed with Omnicef (Cefdinir) ? Aluminum hydroxice Ferrous fumarate Probenecid Bao Tonly Tand If only Tand Hf only All aogce 883. Zemplar(Paricalcitol) is indicated for the treatment of| a Hypertension b ——_Hyperparathyroidism c Diabetes d-—— Glaucoma www.pharmacyexam.com Krisman 884. How many grams of Dextrose are re- quired to prepare 5% of 500cc solution ? a 2.5gm b. 25gm c. 55gm ad Sgm 885. Which of the following is NOT aneu- rotransmitter? a. Epinephrine b. Histamine ©. Dopamine d. MAO 886. How muchAtropine is required to dis- pense | quart of | in 100 solution ? a -9.6mg b. 148gm ce. 2.3mg a 9600mg 887. 20ccof 10% KCI solution, 50.cc of 25% NaHCO solution and 30 cc of 20% CaCI2 are mixed with 5% 1000cc dextrose solution. The infusion should be administered over 8 hours. What is flow rate in drops/min ? (LY. set delivers = 10 drops/ec) a 13.19 b. 14.05 «. 18.01 da 2291 888. Azopt (Brinzolamide) is indicated for the treatment of a Hypertension b Edema ¢ Glaucoma 4 Gout 108 Reference Guide for Foreign Pharmacy Licensing Exam Questions and Answers- 2nd Revised Edition 889. Toprepare 0.25% of 1000 ce Dakin so- lution, how many ce of 5% sodium hypochlorite are required ? a 100cc b. 50ee © 60ee d. 40ce 890. ‘Tazorac (Tazarotene) topical cream isi dicated forthe treatment of 1 Psoriasis I Acne II Poison ivy a Tonly b Land {1 only c iLand I only d All 891. The extended release action of Ditropan XI Oxybutynin ER) is attributed to its a. GITSsystem b. ‘Wax matrix form c Granules formulation d. Inner coat with outer coat 892. The initial concentration of drug decom- position according to first order kinetic is 10? units/ im]. When the concentration of drug falls below 10 units/cc, it should be removed from marke. What would be the expiration date for this prod- uct? [K = 2.303 x 10" hour! } a 20x 10° hours b. 40x 10°hours c. $x 10° hours d. 40 hours 893. Themajoradverse effect of Doxil (Doxo- nubicin)is www.pharmacyexam.com Krisman 1 Cardiac failure I _Liverfailure MI. Renal failure a Tonly b —_TandIonly c Mand Il only dll 894. 1f 500mg of triamcinolone powder are mixed with | Ib of 1% triamcinolone cream, what is the percentage of triamcinolone in the final mix- ture? 1.25 % wiw 3.56 % wiw 2.1% wiw 4.2% wiw apee 895. Which ofthe following drugsis indicated for the treatment of Cisplatin toxicity ? Acetyleysteine Antiplatin ‘Ammifostine Naltrexon 896. Whatis the osmolarity of 500 cc of 25% of sodium bicarbonate solution ? (mw =84 gm/mole) a 5952.38 b. 2763.18 ©. 7032.11 d. 1809.09 897. Clotrimazole Troches are indicated forthe treatment of a. Onychomycosis, b. Oral candidiasis, ©. Athlete’s foot d. Ringworm infection of groin 109 Reference Guide for Foreign Pharmacy Licensing Exam ‘Questions and Answers- 2nd Revised Edition 898. How many calories are provided by 500 cc of D,,W solution ? a 145 calories b. 510 calories ¢. 325calories d.—252calories 899. The disorder of the eye in which the per- son can see near objects perfectly but finds it

You might also like